Download as pdf or txt
Download as pdf or txt
You are on page 1of 56

Critical Reasoning

Content

S. No. Chapter Page Number


1. Understanding the Structure of an Argument 5
2. Identifying Assumptions 8
3. Finding out the Main Point 13
4. Strengthening of an Argument 16
5. Weakening of an Argument 20
6. Understanding Inferences 23
7. Cause and Effect Reasoning 26
8. Resolving Paradox in an Argument 31
9. Method and Flaw in the Reasoning 35
10. Parallel Reasoning 43
11. Practice Questions on CLAT Pattern 46
CLAT CRITICAL REASONING 1. The Stimulus or the Passage: The stimulus is the heart of
The Common Law Entrance Test contains around 24-27 Critical the CR question, presenting a scenario, argument, or passage.
Reasoning Questions to evaluate your ability to understand, It contains information, evidence and conclusions relevant
analyse, and critically assess the given passages, and identify to the argument. Analyzing the stimulus is the first step in
the strengths and weaknesses of the arguments presented. This understanding the context and identifying key elements.
module aims to provide you with the basic concepts of critically 2. The Question Stem: Directs your attention to a specific
evaluating an argument presented and tries to explain the aspect of the argument. It outlines the task you must
application of those concepts with the help of examples. To ace perform—drawing conclusions, identifying assumptions, or
the Critical Reasoning Section in the CLAT Paper, you should evaluating the reasoning. Understanding the question stem
start with understanding and clarifying the basic tenets of an is crucial for focusing your analysis.
argument and then move to the advanced concepts involved in 3. Answer Choices: Four answer choices are given in the CLAT
solving a Critical Reasoning question. To elaborate, the basic paper and you are required to choose the most appropriate
concepts include a comprehensive grasp of the building blocks out of them. It is important to analyse each option carefully
of an argument, identifying the premise, assumption, conclusion, before discarding that option. The process of elimination is
inference etc. The advanced category of questions covers topics generally deployed to reach the correct answer.
such as the method of reasoning, flaws in the argument, paradox,
Illustration
parallel reasoning, cause-effect dynamics, course of action, etc.
After you have covered the concepts of a particular topic, delve Stimulus or Passage: In a small town, there’s an ongoing debate
into practising more and more questions on that topic. Once you about the effectiveness of a proposed curfew for teenagers.
have built some confidence and grab on it, consider moving on to Proponents argue that a curfew will reduce crime rates among
the other topic. Remember, all the concepts in Critical Reasoning teenagers, ensuring safety for both the youth and the community.
are interlinked; therefore, it is suggested that you follow the Opponents counter that a curfew infringes on the freedom of
sequence of topics covered in this module. teenagers and doesn’t address the root causes of delinquency.
To summarise, the following are the points you need to Question Stem
remember while strategising your way ahead with the Critical 1. Which of the following, if true, would most weaken the
Reasoning section: argument in favour of the proposed curfew?
1. Establishing a solid understanding of the foundational Answer Choices
concepts. (a) A recent study indicates that the majority of crimes involving
2. Focus on dissecting complex arguments systematically. teenagers occur during daylight hours.
3. Practice is the cornerstone of mastering Critical Reasoning. (b) The town has experienced a decrease in overall crime rates
4. Regularly reviewing your performance and learning from in the past year.
mistakes. (c) Teenagers in the town actively participate in after-
school programs focused on skill-building and personal
PARTS OF A CRITICAL REASONING development.
QUESTION (d) The proposed curfew is set to begin at 10 PM, impacting
A Critical Reasoning (CR) question typically consists of several teenagers who are generally not involved in criminal
components, each serving a specific purpose in assessing your activities.
ability to analyse and evaluate arguments. Here are the key
elements you might encounter:
1 Understanding the Structure
of an Argument

In this chapter, we will delve into the intricacies of dissecting articulated, while implied conclusions require a deeper level of
arguments, deciphering their components, and comprehending analysis, involving an understanding of the logical connections
the relationships that underpin them. At its core, an argument between statements. To identify conclusions, ask yourself the
is a construction of premises and a conclusion. The premises question; What does the author want you to believe? What points
are statements that lay the groundwork, providing evidence or follow from the above-stated points?
support, while the conclusion is the ultimate point or claim the
author seeks to establish. Let’s begin by first understanding what Illustration
are the pieces of an argument, how they fit together and how we Argument: “The implementation of mandatory community
categorise them. service for high school students is an indispensable measure for
fostering civic responsibility and personal development”.
A. PREMISE Conclusion: “Therefore, introducing mandatory community
All arguments contain at least one premise. A premise is the core service requirements will contribute significantly to shaping
of the argument that supports the conclusion. It can be a fact or responsible and well-rounded individuals”.
an opinion; can be a description, historical information, statistical In this argument, the conclusion is the sentence that
or numerical data, or a comparison of things. It gives the reason begins with “Therefore”. It represents the central claim that the
why conclusions should be accepted. In essence, premises are the author aims to establish: the idea that implementing mandatory
building blocks upon which the entire argument rests. To identify community service for high school students will contribute
premises, ask yourself the question; What reasons has the author significantly to shaping responsible and well-rounded individuals.
used to persuade you? What are the evidences that exist and why The conclusion synthesizes the information presented in the
should you believe in what he says? premises, which could include arguments about the benefits of
community service, the positive impact on personal development,
Illustration
and the cultivation of civic responsibility.
Argument: “Implementing a curfew for teenagers in our town is
To help you out in the process of identifying the premise
necessary to reduce crime rates and ensure the safety of both the
and the conclusion effectively, a list of signal words or indicators
youth and the community”.
is provided below. Following are the words which the author
Premise: “Statistics from the past year show a significant increase generally uses to introduce a premise or a conclusion. Remember
in criminal activities involving teenagers during late-night hours, this is not an exhaustive list but it will help you to categorise and
especially after 10 PM”. break the complex set of arguments presented in a passage. While
In this argument, the premise is the statement providing reading the passages train your eyes to spot these signal words
evidence or support for the conclusion that implementing a curfew and then mark that statement as the premise or conclusion.
is necessary. The premise cites statistical data from the past
year, specifically highlighting an increase in criminal activities Premise Indicators Conclusion Indicators
involving teenagers during late-night hours. This premise aims Since Hence
to convince the audience that the proposed curfew is a necessary
Because Thus
measure to address the identified issue.
For Therefore
B. CONCLUSION In that So
Most but not all arguments contain a conclusion. It represents
Given that Accordingly
the author’s main opinion or claim and can be in the form of
a prediction, a judgment of quality or merit, or a statement of Due to Clearly
causality. The conclusion is the core assertion that the author aims Owing to Conclude that
to persuade the audience to accept. It serves as the culmination
of the premises, drawing together the supporting elements into We know this by Must be that
a singular claim. Conclusions can be explicitly stated or implied This can be seen from Shows that
within the context of the argument. Explicit conclusions are
Additional Premise: “Studies conducted in various industries
For example Follows that
consistently show that companies offering a four-day workweek
For the reason that For this reason experience a significant reduction in employee burnout and an
As indicated by As a result increase in overall job satisfaction”.
Because of In summary In this example, the additional premise strengthens the
argument by providing empirical evidence from studies. It
Considering that To sum up
reinforces the initial claim by introducing a broader perspective,
Assuming that Ultimately suggesting that a four-day workweek is not only a theoretical
concept but has been empirically linked to positive outcomes in
Application Drill terms of employee well-being and satisfaction.
Read the short argument presented and categorise the parts of the
argument as premise or conclusion: Counter Premise
1. “Given that the price of steel is rising, we will no longer be Counter-premises introduce alternative perspectives or challenges
able to offer discounts on our car parts”. to the main argument, encouraging a nuanced evaluation of the
issue at hand. They serve as critical elements that challenge the
Sol. Premise: “Given that the price of steel is rising”. assumptions made by the author, prompting a more thorough
Conclusion: “We will no longer be able to offer discounts examination of the issue. It involves actively seeking out
on our car parts”. potential weaknesses in the argument and considering alternative
2. “Since we need to have many different interests to sustain perspectives that may impact the overall validity of the conclusion.
us, the scientists’ belief must be incorrect”.
Illustration
Sol. Premise: “Owing to the ability of individual warlords to
maintain powerful armed forces”. Argument: “Implementing a four-day workweek will enhance
Conclusion: “The political situation in Somalia is unstable”. employee satisfaction and productivity”.
3. “Television hurts society. This can be seen from the poor Counter-Premise: “However, critics argue that a four-day
school performance of children who watch significant workweek may lead to a decrease in overall productivity due to
amounts of television and from the fact that children who the condensed work hours and potential disruptions in project
watch more than six hours of television a day tend to read timelines”.
less than non-television-watching children”. Here, the counter-premise introduces an opposing
Sol. Premise 1: “This can be seen from the poor school viewpoint, challenging the assumption that a four-day workweek
performance of children who watch significant amounts of unequivocally leads to increased productivity. The counter-
television”. premise forces a more comprehensive analysis of potential
Premise 2: “And from the fact that children who watch drawbacks, such as condensed work hours and potential
more than six hours of television a day tend to read less than disruptions, encouraging a reader or test-taker to consider
non-television watching children”. alternative perspectives and potential limitations to the argument.
Conclusion: “Television hurts society”. Note how this Following is the list of signal words or indicators that will
sentence does not contain a conclusion indicator. Yet, we help you identify an additional premise or a counter premise:
can determine that this is the conclusion because the other
sentence contains two premises. Additional Premise Counter Premise
Indicators Indicators
WHAT IS THE ADDITIONAL PREMISE AND
COUNTER PREMISE? Besides Yet
Moreover But
Additional Premise
Additional premises are supplementary statements that provide Furthermore However
further support to the main argument. They enhance the credibility In addition On the other hand
and persuasiveness of the primary premises, reinforcing the
After all Although
logical connection between evidence and conclusion. These
premises contribute to a more robust and convincing argument What’s more Despite
by addressing potential gaps, anticipating objections, or providing Consider also Whereas
examples to illustrate key points.
Another contributing factor is Contrary to this
Illustration
Not only…..but also Conversely
Argument: “Implementing a four-day workweek will enhance
employee satisfaction and productivity”. In light of this evidence On the flip side

6 Critical Reasoning PW
Application Drill Premise: “A survey conducted among employees in various
Read the short argument presented and categorise the parts of the industries revealed that a majority expressed a desire for
argument. more flexible work arrangements and cited it as a crucial
factor influencing job satisfaction”.
1. During last night’s robbery, the thief was unable to open
Premise: “Research studies consistently show that
the safe. Thus, last night’s robbery was unsuccessful even
organizations with flexible work policies experience lower
though the thief stole several documents. After all, nothing
rates of employee burnout and higher levels of overall job
in those documents was as valuable as the money in the safe. satisfaction”.
Sol. Premise: “During last night’s robbery, the thief was unable Additional Premise: “Moreover, companies that have
to open the safe”. adopted flexible work-from-home policies report increased
Counter-Premise: “Even though the thief stole several employee retention rates and improved recruitment
documents”. outcomes”.
Additional Premise: “After all, nothing in those documents Conclusion: “Therefore, adopting a flexible work-from-
was as valuable as the money in the safe”. home policy is likely to enhance both employee satisfaction
Conclusion: “Thus, last night’s robbery was unsuccessful”. and overall workforce well-being”.
2. An argument has been given and you need to form a premise, Counter-Premise: “However, critics argue that a remote
additional premise, conclusion and counter premise for it. work environment might lead to reduced collaboration
and teamwork, potentially impacting project outcomes and
Argument: “Implementing a flexible work-from-home hindering the organization’s overall productivity”.
policy will contribute to increased employee satisfaction
Note: Repeat the idea of these application drills while reading
and a more favourable work-life balance”.
newspapers, articles, blogs, etc. Break down the available
Sol. You can form your answer. This is an example of a possible information and classify them under these heads. This will increase
correct answer. This exercise will help you clarify your your overall understanding and accuracy while attempting the
fundamental understanding of the basic constituents of an logical reasoning section in the CLAT paper.
argument.

Understanding the Structure of an Argument 7


2 Identifying Assumptions

WHAT IS AN ASSUMPTION? supporting logic explicitly presented in the argument is sufficient


An assumption in critical reasoning is an underlying belief that the to justify the conclusion. If there’s a need for additional evidence
author must accept as true for their argument to hold, even though or logic to make the argument coherent, an assumption is likely
it might not be explicitly stated. It is a foundational element that present. The author often relies on the reader to automatically
bridges the gap between the provided premises and the conclusion, make these assumptions without explicitly stating them. To find
representing a necessary link in the logical chain of reasoning. these gaps or holes, ask yourself if the evidence logically supports
Assumptions are not always universally true in the real world; the argument and if any extra support is required. Once a gap is
their significance lies in the fact that the author relies on them identified, the next step is to articulate the missing evidence or
to make their claim. These unstated premises often represent the supporting logic that needs to be assumed for the argument to
author’s worldview, values or expectations. In other words, you make sense. Consider what assumptions you, as a reader, need to
can also look at assumptions as additional information that the accept for the argument to be valid.
author has assumed the reader already knows while reading the To test the validity of the assumed assumption, attempt to
argument. deny or negate it. If negating the assumption weakens the argument,
The following figure shows the relationship between a the assumption is valid because, without it, the argument loses its
premise, a conclusion and an assumption. The assumption is the coherence. On the other hand, if negating the assumption doesn’t
hidden foundation of a premise. impact the argument’s strength, then the assumption is incorrect.
Conclusion In simple terms, spotting assumptions involves finding the hidden
↑↑↑ beliefs that the author expects you to accept for their argument to
Premise work. These unspoken premises often act as the glue that holds
↓↓↓ the argument together.
Assumption
The Negation Technique
ASSUMPTION QUESTION STEMS
The negation technique is a methodical approach to testing the
The following list of the probable question stems will help you validity of these assumptions.
identify that the questions asked are related to identifying the
assumption in the question.  Step 1: Identify the Assumption: Begin by pinpointing the
assumption in the argument. This is the underlying belief
The argument in the passage depends on which of the following that, while not explicitly stated, is crucial for the argument’s
assumptions? coherence. It often acts as the linchpin holding the reasoning
The argument above assumes that together.
 Step 2: Articulate the Assumption: Clearly state the
The conclusion above is based on which of the following
assumptions? assumption. Express the implicit belief that the author
relies on but doesn’t overtly mention in the argument. This
Which of the following is an assumption made in concluding articulation helps in formulating a clear negation of the
above? assumption.
The conclusion of the argument above cannot be true unless  Step 3: Negate the Assumption: Take the assumption you’ve
which of the following is true? identified and negate it. In simple terms, express the opposite
or deny the truth of the assumption. This step is pivotal in
HOW TO SPOT ASSUMPTIONS IN A CRITICAL testing whether the argument hinges on this particular belief.
REASONING QUESTION?  Step 4: Assess the Impact: Now, critically examine how the
Spotting assumptions is a crucial skill in critical reasoning, and negation of the assumption influences the overall strength of
it involves identifying gaps or missing links in an argument. To the argument. Ask yourself whether the argument still holds
discern assumptions, examine whether the provided evidence or its ground or if it falters without the assumed premise.
 Step 5: Two Possible Outcomes: There are two potential compelling case for implementing a flexible work schedule and
outcomes: expecting increased satisfaction.
• If negating the assumption significantly weakens
Illustration
the argument, the assumption is crucial. Your initial
identification is valid and the argument relies on this Assumption: Regular exercise contributes to weight loss.
unstated belief. Negation: Regular exercise does not contribute to weight loss.
• If negating the assumption doesn’t impact the argument’s Assessment: If the argument significantly weakens without the
strength, the assumption may not be as indispensable as assumption, it implies that the belief in exercise leading to weight
initially thought. The argument can maintain its cogency loss is crucial. However, if the argument remains robust even
without this particular implicit premise. when negating this assumption, it suggests that the connection
Most of the CLAT Critical Reasoning Questions based between exercise and weight loss might not be as pivotal as
on assumptions can be easily solved by deploying this negation initially thought.
technique. The task is to first understand all the steps of how to
apply the negation technique and then practice as many questions Application Drill
as you can to strengthen your grab on the concept of assumption. Argument: “Introducing a recycling program in the office will
Refer to the following Illustrations to get more clarity on the significantly reduce the company’s carbon footprint”.
application of assumptions and then solve the application drills
that follow. 1. Which of the following is the assumption of the argument?
(a) Employees are environmentally conscious and willing
Illustration to participate in recycling efforts.
Argument: “Implementing a flexible work schedule will lead to (b) The office generates a substantial amount of recyclable
increased employee satisfaction”. waste.
Assumption: Employees value flexibility in their work schedules. (c) The cost of implementing the recycling program is
In this argument, the assumption is the unstated belief that minimal.
forms the linchpin connecting the proposed action (implementing (d) The company’s competitors have successfully
a flexible work schedule) to the desired outcome (increased
implemented recycling programs.
employee satisfaction). To analyze this assumption more
comprehensively, let’s break down the steps: Sol. Option (a) (Correct Answer): The assumption underlying
 Step 1: Identify the Assumption: The assumption here is the argument is that employees are environmentally
that employees value flexibility in their work schedules. This conscious and willing to participate in recycling efforts. If
belief isn’t explicitly mentioned in the argument but is crucial employees are not environmentally conscious or unwilling
for the argument’s logic to hold. to participate, the effectiveness of the recycling program is
 Step 2: Articulate the Assumption: Express the assumption jeopardized. Therefore, Option (a) is the correct answer.
explicitly: “Employees value flexibility in their work Option (b) (Incorrect): While it might seem logical that
schedules”. a recycling program is effective if there’s a substantial
 Step 3: Negate the Assumption: Negate the assumption: amount of recyclable waste, the argument does not depend
“Employees do not value flexibility in their work schedules”. on the quantity alone. The assumption is about employee
 Step 4: Assess the Impact: Now, consider how the argument behaviour and willingness to participate, making Option (b)
would fare if employees did not value flexibility. If the less relevant to the logical bridge needed in the argument.
argument weakens significantly without this assumption, it Option (c) (Incorrect): The cost of implementing the
means that the initial assumption is crucial. recycling program, while potentially important, is not a
 Step 5: Answer Explanation: If the argument’s necessary assumption for the argument. The argument
persuasiveness relies heavily on the assumption that focuses on the reduction of the company’s carbon footprint,
employees value flexibility, then the belief in the importance and the assumption is more about employee behaviour than
of flexible work schedules is fundamental to the conclusion. the program cost. Therefore, Option (c) is not as crucial to
The assumption acts as a bridge, connecting the proposed the argument.
action with the expected outcome. If employees do not value
flexibility, the argument loses its logical foundation. Option (d) (Incorrect): The success or failure of the
company’s competitors in implementing recycling programs
Final Answer is not a necessary assumption for the effectiveness of the
The assumption, therefore, is that “Employees value flexibility in proposed recycling program in this specific company. The
their work schedules”. This implicit belief, when made explicit, argument is centred around the company’s situation, and the
ensures the logical coherence of the argument. It underscores the success of competitors is tangential to the logical connection
necessity of understanding the employees’ preferences to make a needed for the conclusion.

Identifying Assumptions 9
UNDERSTANDING IMPLICIT AND EXPLICIT Defender Assumption: “The employees are adequately trained
ASSUMPTIONS to use the new software system”.
In this scenario, the assumption acts as a defender by
Explicit Assumptions
assuming that employees are well-trained to use the new software
Explicit assumptions are the beliefs that are clearly stated or system. If this assumption is not valid, the argument becomes
expressed in the argument. These are the premises that the author vulnerable to the counterargument that insufficient training might
directly puts forward to support their conclusion. For example, hinder the effectiveness of the new software, thereby potentially
in the argument “Increasing funding for education will improve reducing workplace efficiency.
student performance”, the explicit assumption might be stated
as “More resources lead to better educational outcomes”. These Application Drill
assumptions are transparent and can be easily identified within 1. Try to solve the following question on assumption. Verify
the argument itself. your line of reasoning with the help of the explanation
provided at the end of the question.
Implicit Assumptions
For several years, Nighttime News attracted fewer viewers
Implicit assumptions, on the other hand, are beliefs that are not
than World News, which broadcasts its show at the same time
explicitly stated in the argument but are necessary for the argument
as Nighttime News. Recently, the producers of Nighttime
to make sense. These assumptions are often hidden beneath the
News added personal interest stories and increased coverage
surface and need to be inferred based on the logical structure of
of sports and weather. The two programs now have a roughly
the argument. Going back to our education example, an implicit
equal number of viewers. The recent programming changes
assumption could be “Improved student performance is directly
persuaded viewers to switch from World News to Nighttime
linked to increased funding”. Even though not directly mentioned,
News.
this assumption is crucial for the conclusion to be valid.
The conclusion above is properly drawn if which of the
WHAT ARE SUPPORTER AND DEFENDER following is assumed?
ASSUMPTIONS? (a) Viewers are more interested in sports and weather than
in personal interest stories.
Supporter Assumption (b) The programming content of Nighttime News is more
A supporter assumption is an underlying belief or premise that closely aligned with the interests of the overall audience
strengthens and reinforces the argument’s conclusion. It is an than is the content of World News.
assumption that, if true, makes the conclusion more credible and (c) Some World News viewers liked the new Nighttime
likely. Essentially, a supporter assumption helps bridge the gap News programming better than they liked the World
between the provided evidence or premises and the conclusion, News programming.
enhancing the overall persuasiveness of the argument.
(d) There are other possible causes for an increase in the
Illustration number of viewers of Nighttime News, including a
Argument: “Increasing funding for public libraries will improve recent ad campaign that aired on many local affiliates.
literacy rates in the community”. Sol. Option (a): This statement, even if true, does not necessarily
Supporter Assumption: “Access to a well-funded library system support the conclusion. The argument focuses on the overall
positively impacts people’s reading habits and, consequently, increase in Nighttime News viewership, not the relative
improves literacy rates”. interest in specific content within Nighttime News.
In this case, the assumption is that a well-funded library Option (b): This assumption aligns with the conclusion. If
system positively influences people’s reading habits, thereby Nighttime News is more aligned with the overall audience’s
contributing to improved literacy rates. If this assumption holds, it interests, the recent programming changes could indeed have
fortifies the argument that increasing funding for public libraries persuaded viewers to switch from World News to Nighttime
will lead to enhanced literacy rates. News.
Option (c): This assumption is critical for the conclusion.
Defender Assumption If some World News viewers preferred the new Nighttime
A defender assumption is an implicit belief that protects the News programming, it supports the idea that the recent
argument from potential counterarguments or challenges. It changes led to a shift in viewership.
is an assumption that, if negated or proven false, weakens the Option (d): This introduces an alternative cause for the
argument’s strength. A defender assumption is often essential for increase in Nighttime News viewership. If true, it weakens
shoring up the argument against possible criticisms. the argument, suggesting that factors other than the
programming changes may be responsible for the shift in
Illustration viewership.
Argument: “Implementing a new software system will increase The correct answer is (c): Some World News viewers liked
workplace efficiency”. the new Nighttime News programming better than they liked

10 Critical Reasoning PW
the World News programming. This assumption strengthens conditions. In simple terms, a conditional statement is comprised
the argument by providing a direct link between the of events or circumstances where the occurrence of one condition
programming changes and the observed shift in viewership. (sufficient) indicates that another condition (necessary) must also
If some viewers prefer the new Nighttime News content, it occur. Any conditional statement consists of at least one sufficient
supports the conclusion that the recent programming changes condition and at least one necessary condition.
persuaded viewers to switch from World News to Nighttime
News. Sufficient Condition
2. Country N’s oil production is insufficient to meet domestic A sufficient condition is an event or circumstance whose occurrence
demand. To sharply reduce its dependence on foreign guarantees that a necessary condition will also occur. It serves as
sources of oil, Country N recently embarked on a program a trigger, signalling that the necessary condition is assured. Think
requiring all of its automobiles to run on ethanol in addition of it as the key that unlocks the door to the necessary condition. If
to gasoline. With its oil production, Country N produces the sufficient condition happens, you can be certain the necessary
enough ethanol from agricultural by-products to meet its condition follows.
current energy demand. Necessary Condition
Which of the following must be assumed to conclude that On the flip side, a necessary condition is an event or circumstance
Country N will succeed in its plan to reduce its dependence that must happen for a sufficient condition to occur. It is a
on foreign oil? prerequisite, without which the sufficient condition cannot take
(a) Electric power is not a superior alternative to ethanol in place. While the presence of the necessary condition ensures the
supplementing automobile gasoline consumption. sufficient condition is possible, its absence prevents the sufficient
(b) In Country N, domestic production of ethanol is condition from happening.
increasing more quickly than domestic oil production.
Example
(c) Ethanol is suitable for the heating of homes and other
applications aside from automobiles. 1. Unlocking a Door: Sufficient Condition (Having the Key):
If you have the key, that’s enough to unlock the door. So,
(d) Ethanol is as efficient as gasoline in terms of mileage
having the key is a sufficient condition for unlocking the door.
per gallon when used as fuel for automobiles.
For example, if you possess the key to your house, you can
Sol. Option (a): This statement is irrelevant to the plan’s success use it to unlock the door and get inside.
in reducing dependence on foreign oil. The argument Necessary Condition (Using the Key): On the flip side,
specifically focuses on the use of ethanol in addition to to unlock the door, you must use the key. Using the key is
gasoline in automobiles, not the comparison with electric necessary; without it, the door stays locked. For instance,
power. having the key might not be enough. You have to insert it
Option (b): This assumption is crucial for the success of the into the lock and turn it to unlock the door.
plan. If domestic ethanol production is not increasing at a 2. Making Tea: Sufficient Condition (Boiling Water): If you
pace greater than domestic oil production, it could hinder have boiling water, that’s sufficient to make tea. Boiling water
the goal of reducing dependence on foreign oil. is a sufficient condition for making tea. For example, if you
Option (c): This information is interesting but not necessary have a kettle on the stove with water reaching a boil, you’re
for the conclusion about reducing dependence on foreign all set to make tea.
oil. The argument is centred around the use of ethanol in Necessary Condition (Using Tea Leaves): However,
automobiles, not in other applications. to make tea, you must use tea leaves. Using tea leaves is
Option (d): This assumption is critical for the success of necessary; without them, you can’t make tea. If you only
the plan. If ethanol is not as efficient as gasoline in terms of have boiling water but no tea leaves, you won’t have tea.
mileage per gallon, it could deter consumers from adopting 3. Sending Mails: Sufficient Condition (Having an Email
the plan, impacting the goal of reducing dependence on Account): If you have an email account, that’s sufficient to
foreign oil.
send emails. Having an email account is a sufficient condition
The correct answer is (d): Ethanol is as efficient as gasoline for sending emails. For instance, if you’ve signed up for an
in terms of mileage per gallon when used as fuel for email service, you can send emails.
automobiles. This assumption is necessary for the success
Necessary Condition (Entering Recipient’s Email
of the plan, as it ensures that the switch to ethanol does not
Address): Yet, to send an email, you must enter the recipient’s
lead to a significant decrease in fuel efficiency, thereby
email address. Entering the recipient’s email address is
encouraging the adoption of the plan and contributing to
necessary; without it, your email won’t reach anyone. Even
the reduction in dependence on foreign oil.
if you have an email account, without specifying who you’re
Conditional Reasoning sending it to, the email can’t be sent.
Conditional reasoning is a fundamental aspect of logic that The concept of conditional reasoning does not appear
involves relationships between sufficient and necessary directly in the CLAT paper but when appearing for Other

Identifying Assumptions 11
Law Entrances, the understanding of conditional reasoning In this case, option (d) correctly captures the essence of the
helps immensely in solving advanced categories of Critical conditional statement. It states that if it’s not raining (absence
Reasoning questions as well as Reading Comprehension of the sufficient condition), Sarah won’t bring an umbrella
passages. Generally, the application of conditional reasoning (absence of the necessary condition).
can be seen directly in the ‘Must be True’ type questions. (a) Suggests Sarah always brings an umbrella, even if it
Three Logical Features of Conditional Reasoning doesn’t rain. This is not a valid conclusion based on the
given statement.
1. Sufficient does not cause necessity: The sufficient condition
in a conditional statement does not actively cause the (b) Reverses the conditional relationship, stating that if
necessary condition to happen. Unlike causal reasoning, Sarah brings an umbrella, it must be raining. This
where one event directly influences another, in conditional contradicts the original statement.
reasoning, the sufficient condition is a signal or indicator that (c) Claims that Sarah never brings an umbrella when it rains,
the necessary condition will, is, or has already occurred. which is not supported by the given statement.
2. Temporal flexibility: Temporally speaking, either condition Thus, Option (d) is the correct conclusion as it accurately
can occur first, or the two conditions can occur at the same reflects the conditional relationship established in the
time. original statement. If it’s not raining, Sarah won’t bring an
umbrella, maintaining the logic of conditional reasoning.
3. Not necessarily reflecting reality: The conditional
relationship presented by the author doesn’t have to mirror 2. Statement: “If Max practices piano for at least two hours
reality. Understanding that the author’s statement may not every day, then he will perform exceptionally well at the
align with what seems intuitively reasonable is essential. recital”.
Which of the following conclusions can be validly drawn
Indicator Words based on the given conditional statement?
The following is the list of signal words or indicators that will (a) If Max performs exceptionally well at the recital, he
help you identify a necessary or a sufficient condition: must have practised piano for at least two hours every
To introduce a sufficient To introduce a necessary day.
condition condition (b) Max practices piano for at least two hours every day
only when there is a recital.
If Then
(c) Max never performs well at the recital unless he practices
When Only piano for at least two hours every day.
Whenever Only if (d) If Max doesn’t practice piano for at least two hours every
day, he won’t perform exceptionally well at the recital.
Any Unless
Sol. The given statement establishes a conditional relationship
All Required
where practising piano for at least two hours every day is the
Every Must sufficient condition, and performing exceptionally well at
the recital is the necessary condition. The correct conclusion
Each Except
should align with the nature of conditional reasoning.
To Until In this case, option (d) accurately captures the essence of
People who Without the conditional statement. It states that if Max doesn’t meet
the sufficient condition (not practising for at least two hours
Application Drill every day), he won’t fulfil the necessary condition (not
Try to solve the questions provided below and verify your performing exceptionally well at the recital).
reasoning with the help of explanations that follow: (a) reverses the conditional relationship, suggesting that
1. Statement: “If it rains, then Sarah will bring an umbrella”. practising for at least two hours every day is necessary
for exceptional performance. This contradicts the
Which of the following is a valid conclusion based on the
original statement.
conditional statement?
(b) implies that Max practices piano only when there is a
(a) Sarah always brings an umbrella, even if it doesn’t rain.
recital, which is not supported by the given statement.
(b) If Sarah brings an umbrella, it must be raining.
(c) suggests that Max never performs well unless he
(c) Sarah never brings an umbrella when it rains. practices every day, but this extreme claim is not
(d) If it’s not raining, Sarah won’t bring an umbrella. warranted by the original statement.
Sol. The original statement is “If it rains, then Sarah will bring an Thus, Option (d) is the correct conclusion as it accurately
umbrella”. This sets up a conditional relationship where the reflects the conditional relationship established in the
occurrence of rain is the sufficient condition, and bringing an original statement. If Max doesn’t meet the sufficient
umbrella is the necessary condition. The correct conclusion condition, he won’t achieve the necessary condition, aligning
should align with the nature of conditional reasoning. with the logic of conditional reasoning.

12 Critical Reasoning PW
3 Finding out the Main Point

Understanding the main point is like finding the most important 1. Firstly, it’s crucial to grasp the structure of the argument.
idea in what you’re reading or listening to. It’s like the main Arguments typically consist of premises and conclusions.
message or the key reason behind why someone wrote or spoke Premises are the supporting statements or evidence, while
something. Imagine it as the big idea that everything else connects the conclusion is the main idea that the author wants us to
to, making it super important for us to identify. The main point accept. Paying attention to these elements helps us see the
is like the main idea, the core message that the author or speaker logical flow of the argument.
wants us to get. It’s the main thing they are talking about, the 2. Next, we need to focus on the language used by the author.
central point that ties everything together. Think of it as the main Keywords and phrases often signal the main point. Look for
focus or the big picture that we need to catch onto while reading terms like “therefore”, “thus” or “consequently” as these
or listening. Knowing the main point is like having a guide that usually precede the conclusion. Understanding how different
helps us understand things better. It’s crucial because it guides parts of the argument are connected helps us pinpoint the
us through what we’re reading or hearing, making everything central message.
clearer. It’s like a map that shows us the way, making it easier 3. Now, consider the author’s perspective and purpose.
for us to make sense of the information and think more carefully. Understanding why someone is making an argument helps us
Finding the main point changes depending on what we are see the main point more clearly. Authors may aim to persuade,
reading. In stories, it might be the main theme or the big message inform, or entertain, and recognizing their goal provides
the author wants us to remember. In non-fiction like news or insight into the primary message they want to convey.
articles, the main point is often the writer’s main argument or the
4. Examine the supporting evidence provided in the argument.
key information they’re telling us.
While the main point is the central idea, the supporting
details and evidence strengthen the argument. Analyzing
MAIN POINT QUESTION STEMS
this evidence helps confirm and clarify the main point. It’s
Here are some common question stems that might be used for like understanding why the author believes their main idea
main point questions: is true.
Which of the following best summarizes the central idea of the 5. Look for the counterarguments or opposing viewpoints.
passage/paragraph/argument? Sometimes, authors present contrasting ideas to strengthen
their argument. Identifying these counterarguments can assist
What is the primary purpose of the passage?
in understanding the main point more comprehensively. It’s
The author’s main point in the passage is best described as... like exploring the boundaries of the argument to see where
What does the author primarily want to convey in the passage? it stands strongest.
6. Also consider the tone and style of the argument. The way
Which of the following statements expresses the central focus an author presents their ideas can reveal the emotional
of the paragraph? or persuasive aspects of the main point. Recognizing the
What is the key takeaway from the argument presented in the emotional undertones or the level of persuasion helps us
passage? understand the author’s intended impact.
The passage is primarily concerned with... 7. Lastly, summarize the argument in your own words. Creating
a concise summary forces us to distil the main point from the
What is the author’s primary stance in the passage? supporting details. We’ve likely identified the main point if
How to Identify the Main Point in an Argument? we can express the argument’s essence in a few sentences.
This step is like putting the puzzle pieces together to reveal
Identifying the main point in an argument is like uncovering the
the big picture.
heart of what someone is trying to convince us of. It involves
looking closely at the argument’s structure, understanding the Illustration
core idea, and recognizing why the author or speaker is presenting Argument: Regular exercise is crucial for maintaining good
it. Let us try to understand how to identify the main point in an health. Studies consistently show that individuals who engage
argument: in at least 30 minutes of moderate-intensity exercise five times
a week experience lower rates of chronic diseases, such as heart vehicle market. Embracing EVs is not just an environmental
disease and diabetes. Additionally, exercise has been linked necessity; it’s a strategic move that fosters economic resilience
to improved mental health, reducing the risk of conditions like and technological advancement.
depression and anxiety. Therefore, incorporating regular exercise Critics argue about the limitations of current EV technology,
into one’s routine is a key factor in promoting overall well-being. including range anxiety and the environmental impact of battery
Steps to Identify the Main Point production. However, these challenges are not insurmountable.
Continuous research and development are addressing battery
1. Structure Analysis: Recognize the structure of the argument.
efficiency, charging infrastructure, and recycling methods.
Here, the premises are the benefits of regular exercise
Moreover, the environmental impact of manufacturing traditional
mentioned in the studies, and the conclusion is the assertion
vehicles far outweighs the challenges posed by electric vehicle
that incorporating exercise promotes overall well-being.
production. In this era of conscious consumerism, individuals are
2. Language Clues: Look for language that signals the main increasingly seeking sustainable alternatives. Electric vehicles
point. The word “therefore” in the last sentence indicates the offer not only a green option for personal transportation but also
transition to the conclusion, emphasizing the importance of
significant long-term cost savings. Governments should incentivize
exercise for well-being.
the adoption of EVs, ensuring that cost barriers are minimized and
3. Author’s Perspective and Purpose: Consider why the that citizens are encouraged to make eco-conscious choices.
author is making this argument. In this case, the author is
presenting evidence to persuade readers of the positive impact 1. What is the primary message of the editorial piece?
of exercise on health and well-being. (a) The challenges of electric vehicle technology outweigh
4. Supporting Evidence: Examine the supporting evidence, the potential benefits.
such as the mention of studies linking exercise to lower (b) The transition to electric vehicles is crucial for addressing
rates of chronic diseases and improved mental health. This climate change and offers economic opportunities.
evidence reinforces the main point that exercise is beneficial (c) Electric vehicles are not a practical option for personal
for overall health. transportation due to range limitations.
5. Counterarguments: Identify any counterarguments or (d) Governments should not intervene in promoting electric
opposing viewpoints. In this example, the author doesn’t vehicles; it should be a consumer-driven shift.
present opposing views but rather focuses on reinforcing the
Sol.
benefits of exercise.
(a) Incorrect: The editorial emphasizes that the potential
6. Tone and Style: Observe the tone and style used.
benefits of electric vehicles outweigh the challenges,
The argument employs a factual and informative tone,
so this option is not the main idea.
emphasizing the credibility of the presented studies to
strengthen the persuasive impact. (b) Correct: The main idea of the editorial is that
transitioning to electric vehicles is crucial for addressing
7. Summarize in your Own Words: Summarize the argument
climate change and offers economic opportunities. It
in your own words. The main point is that incorporating
highlights the environmental and economic benefits of
regular exercise into one’s routine is a crucial factor in
embracing EVs.
promoting overall well-being due to the proven health
benefits mentioned in the studies. (c) Incorrect: The editorial acknowledges challenges but
argues that they are not insurmountable, emphasizing
Application Drill the overall benefits of electric vehicles.
Read the following passages and try to identify the main point (d) Incorrect: The editorial suggests that government
of the passage. To verify the answer, consult the explanation that intervention is crucial in incentivizing the adoption of
follows: electric vehicles, making this option inconsistent with
Passage 1 the main idea.
As we stand on the brink of a critical juncture in addressing Passage 2
climate change, we must reevaluate our choices in transportation. Many patients are hesitant to seek second opinions when making
The transition to electric vehicles (EVs) presents an unparalleled decisions about their health, even when considering major
opportunity to curb emissions, reduce our dependence on fossil medical procedures. This hesitation is sometimes based on a lack
fuels, and pave the way for a sustainable future. Governments, of familiarity with a relatively new physician, but even where
industries, and individuals must collaboratively embark on this a strong relationship has been developed between doctor and
journey toward embracing EVs, ensuring a cleaner and greener patient, the person being treated often perceives the interest in
tomorrow. The shift to electric vehicles not only aligns with global a second opinion as an affront to the doctor who has provided
environmental goals but also offers a chance for economic growth the first opinion. This tendency is rather unfortunate, given the
and innovation. Countries investing in EV infrastructure and potential benefits, either of further confirmation that a particular
manufacturing are not only reducing their carbon footprint but are path represents the proper course or of contrary perspectives
also positioning themselves as leaders in the burgeoning electric which can be considered for more fully informed decisions.

14 Critical Reasoning PW
2. Which of the following best represents the main point of the (b) Incorrect: While the passage mentions that some patients
passage above? perceive the interest in a second opinion as an affront
(a) Patients should seek second opinions only in cases of to the initial doctor, it does not generalize this to all
questionable intent on the part of the physician. doctors. The main point is about patient hesitation, not
(b) Some doctors consider the request for a second opinion the reaction of doctors.
offensive. (c) Incorrect: The passage does not claim doctors
(c) Doctors who tell patients not to seek second opinions are telling patients not to seek second opinions to avoid
attempting to avoid competition with other physicians. competition with other physicians. It is more about
(d) Many patients are hesitant to seek second opinions when patient perceptions and behaviours.
making decisions about their health. (d) Correct: This is the correct answer. The passage
Sol. emphasizes the hesitancy of many patients in seeking
second opinions when making health decisions,
(a) Incorrect: The passage does not suggest that patients
should seek second opinions only in cases of questionable highlighting the reasons behind this reluctance.
intent on the part of the physician. The focus is on patient
hesitancy in seeking second opinions for various reasons.

Finding out the Main Point 15


4 Strengthening of
an Argument

At the heart of every argument is a central idea, much like the Argument: “Eating more fruits is good for health”.
main message you want to share. Strengthening an argument is Strengthen: “Studies show people who eat more fruits have
about making that core idea even more solid, giving it a firm fewer health issues”.
foundation so that others are more likely to believe or agree with
Assumption: “Fruits provide essential nutrients that contribute
what you’re saying. strengthening an argument is like adding to good health”.
sturdy pillars to a building to make it more robust and convincing.
Imagine your argument as a bridge that needs to carry the weight Thus, in strengthening, we add more evidence. In assumption,
of your ideas across to others. Strengthening that bridge involves we find the hidden, unsaid things that make the argument complete.
So, strengthening is like adding bricks to a house, while the
reinforcing it with strong materials, making sure it won’t wobble
assumption is finding the secret ingredient that makes it work!
or break when others examine it. Think of your argument like
a puzzle, where every piece plays a role. Strengthening comes Strengthen Question Stems
into play when you notice some gaps or missing pieces. It’s about Here are some common question stems for strengthen questions:
finding those gaps and putting in additional bits of information
or support to make your argument whole and complete. A strong “Which of the following, if true, most strengthens the
argument is like a shield, defending itself against doubts and argument?”
questions. Strengthening involves adding layers of protection “Which of the following, if true, most strongly supports the
by providing extra evidence, clarifying any assumptions, and statement?
addressing possible doubts that someone might have. The aim is
to make your argument so sturdy that it’s hard to poke holes in. “Which of the following, if true, does most to justify the
conclusion above?”
THE DIFFERENCE BETWEEN “Each of the following, if true, supports the claim above
STRENGTHENING AND ASSUMPTION EXCEPT:”
QUESTIONS The author’s argument would be most reinforced by which of
Imagine your argument as a house. Strengthening is like adding the following?
more bricks to make that house sturdier. The question asks, “Can
The argument assumes which of the following to be true for its
you make the argument stronger?” It’s like giving your house
conclusion to be valid?
more walls, making it more convincing. The answer could be
a small addition (just a window), or a big one (an entire new What fact, if confirmed, would best bolster the argument?
room). If it makes the argument even a tiny bit better, it’s a win! Which of the following, if added to the argument, would make
Assumption questions, on the other hand, are like looking for the conclusion more likely to be true?
the hidden superhero behind the scenes. They want to know,
“What must be true for this argument to make sense?” Imagine How to Strengthen an Argument?
your argument is a car, and the assumption is the fuel it runs Let us try to understand a step-by-step approach to solving a
on. Without that fuel (assumption), the car (argument) won’t go strengthening question and explore how to do this effectively.
anywhere. Assumptions are the silent helpers, the things we don’t  Step 1: Identify the Conclusion: Imagine the argument as
say out loud but need for our argument to be true. a journey, and the conclusion is the destination. Your job is
In simple terms, Strengthening is about making the to make that destination more reachable. Ask, “Does this
argument more convincing, adding whatever helps, big or small. answer choice help the author reach their destination?” If
The assumption is finding the missing pieces, the things we don’t yes, you’ve found the right answer.
say but need for our argument to be true. Strengthen questions  Step 2: Personalize the Argument: Make the argument
ask you to support the argument in any way possible. Assumption your own. Put yourself in the shoes of the author and think,
questions ask you to identify a statement that the argument “How would this be true in my world?” Personalizing helps
assumes or supposes. Let us try to gain more clarity on this aspect you connect with the argument, making it easier to see what
by understanding the following example: could make it stronger.
 Step 3: Look for Weaknesses: Even superheroes have 3. Out-of-Scope Answers: Imagine discussing the benefits
weaknesses, and so do arguments. If you spot a crack or of running, and someone suddenly talks about the latest
hole in the argument, find an answer choice that acts like fashion trends. Out of Scope answers miss the point entirely,
a patch. It’s about closing gaps and making the argument supporting ideas unrelated or only vaguely connected to the
more airtight. For example: If someone says, “I always have argument. For example: If the argument is about “the impact
good weather on my vacations”, but forgets to mention they of technology on education”, an out-of-scope answer might
only go to tropical places, the weakness is not considering talk about “technological advancements in medicine”.
different destinations. An answer that says, “They always go These pitfalls are like shiny distractions on the path to the
to tropical places” would strengthen the argument.
correct answer. Opposite answers mislead, Shell Game answers
 Step 4: Identify Missing Links: Arguments are like chains; play tricks, and Out of Scope answers take you on a detour. Keep
if a link is missing, the chain weakens. Find those missing an eye out for these traps, and you’ll be better equipped to choose
links, often assumptions made by the author. Bringing the answer that genuinely strengthens the argument.
these assumptions to light makes the argument stronger by
confirming the author’s thinking. Illustration
 Step 5:Validate Analogies and Surveys: If the argument 1. Consumer Advocate: Many household cleaners contain
uses analogies (comparing things) or surveys, those ingredients which are highly toxic when ingested by children
comparisons need to be valid. Look for answer choices that or pets. Because of this significant risk, I propose a law
support the validity of these comparisons. It’s like making prohibiting the use of such toxic ingredients in household
sure the foundation of a building is solid before adding more cleaners. Which of the following, if true, provides the most
floors. support for the argument above?
 Step 6: Varying Degrees of Strength: Remember, the right (a) Most toxic household cleaners have labels which clearly
answer doesn’t always have to make the argument super warn of their toxicity.
strong. It could be a little boost or a major power-up. This
(b) There are many different types of household cleaners,
flexibility makes these questions tricky. The correct answer
and some are more 209 effective than others.
is the one that moves the argument in the right direction, no
matter how big or small the step. (c) When the use of household cleaners is discontinued,
So we can conclude that strengthening arguments is harmful bacteria are more likely to propagate in areas
about reinforcing the logic, closing gaps, and adding supportive where children and pets are commonly found.
elements. It’s the art of turning a good argument into a great one, (d) The toxic ingredients in most household cleaners could
making it harder for doubts to creep in. be replaced by comparably priced, non-toxic ingredients
of equal or better quality.
Incorrect Answer Traps or Pitfalls in Strengthen
Questions Sol. Argument Recap: The consumer advocate argues for a law
Navigating through Strengthen questions is like avoiding hidden prohibiting the use of toxic ingredients in household cleaners
traps. Let us uncover the common pitfalls to steer clear of when due to the risk they pose when ingested by children or pets.
seeking the right answer: To strengthen this argument, we need to find an answer
1. Opposite Answers: Imagine you’re trying to build a bridge, choice that provides additional support for the proposed law.
but someone hands you materials that will make it collapse Option (a): This choice does not support the argument
instead. Opposite answers do just that—they weaken the because warning labels, while important for awareness, do
argument instead of strengthening it. They may seem not eliminate the risk presented by toxic ingredients.
attractive because they’re closely tied to the conclusion, but Option (b): This choice is irrelevant to the argument as
they lead you in the wrong direction. For Example: If the effectiveness does not address the safety concern raised by
argument says, “Eating more fruits is healthy”, an opposite the consumer advocate.
answer might say, “Studies show people who eat more fruits Option (c): This weakens the argument by suggesting a
are less healthy”.
potential negative consequence of discontinuing household
2. Shell Game Answers: Picture a game of hide and seek where cleaners. It does not support the proposed law.
the answer hides behind a shell. Shell Game answers present
Option (d): This is a strong contender. It supports the
an idea similar to what’s in the argument, but with a subtle
argument by suggesting a feasible alternative that maintains
twist that makes it incorrect. It’s like looking for your keys,
the quality of household cleaners while eliminating toxicity.
finding something similar, but it doesn’t open the door. For
Example: If the argument talks about “increased exercise Option (d) provides the most support for the argument,
leads to weight loss”, a Shell Game answer might say, “More proposing a solution that aligns with the consumer advocate’s
movement is linked to shedding pounds”, subtly changing goal of safer household cleaners without compromising
the concept. quality.

Strengthening of an Argument 17
Causality and Strengthen Questions Illustration
Because Strengthen and Weaken questions require you to perform Among the three 24-hour pharmacies in the city, Sonny’s
opposite tasks, to strengthen a causal conclusion you take the Pharmacy is consistently the 212 most profitable. Sonny claims
exact opposite approach that you would in a Weaken question. that since the three pharmacies carry the same products, the store’s
Strengthening causal conclusions is like building a fortress around success is attributable to its superior customer service.
the cause-and-effect relationship. Unlike weakening, where we 2. Which of the following, if true, most strongly supports
look for weak spots, here, we reinforce the link between cause
Sonny’s claim?
and effect. Let us understand the steps involved in solving this
type of question: (a) The other two pharmacies in town advertise less than
Sonny’s.
1. Identify the Causal Relationship: Start by understanding
(b) The other two pharmacies’ products are sold for
the cause-and-effect relationship presented in the argument.
Pinpoint what the author claims is the cause leading to approximately the same prices as similar products at
a specific effect. For example: In an argument stating, Sonny’s.
“Increased exercise leads to weight loss”, identify exercise (c) Sonny’s is near the centre of the city, a location that is
as the cause and weight loss as the effect. convenient for most of the city’s pharmacy customers.
2. Eliminate Alternate Causes: Look for any potential (d) The three pharmacies in town often require different
alternative causes that could also explain the effect. If the wait times for the same prescription.
argument asserts a specific cause, showing that other potential Sol. Argument Recap: Sonny’s Pharmacy claims its consistent
causes are unlikely strengthens the case for the stated cause. profitability is due to superior customer service compared
For example: If arguing that “Increased exercise leads to to the other two 24-hour pharmacies in the city.
weight loss”, consider and dismiss alternative causes like
To strengthen Sonny’s claim, we need an answer choice that
diet changes or metabolism shifts.
aligns with the idea that Sonny’s superior customer service
3. Demonstrate Cause Leads to Effect: Provide examples is indeed contributing to its success.
or evidence showing that when the cause occurs, the effect
Option (a): This doesn’t directly relate to customer service,
consistently follows. This reinforces the idea that the stated
which is the focus of the argument. Advertising differences
cause is indeed linked to the claimed effect. For example,
Offer instances where individuals increased their exercise, might not necessarily explain Sonny’s superior profitability.
resulting in noticeable weight loss. Option (b): Again, this doesn’t address customer service.
4. Show Absence of Cause, Absence of Effect: Illustrate Similar pricing doesn’t necessarily mean similar service
scenarios where the absence of the cause corresponds to quality.
the absence of the effect. This strengthens the argument by Option (c): This is a strong contender. It suggests that
emphasizing that the presence of the cause is necessary for Sonny’s prime location might be contributing to its success,
the effect to occur. Example: Highlight situations where supporting the claim that customers find it convenient,
individuals who did not increase their exercise did not possibly due to superior service.
experience significant weight loss. Option (d): This is relevant to customer service. If Sonny’s
5. Eliminate Reversed Relationship Possibility: Rule out consistently has shorter wait times, it supports the idea that
the possibility that the effect could be causing the cause, superior service contributes to its success.
confirming the intended direction of the cause-and-effect Thus, Option (d) provides the most direct support for Sonny’s
relationship. Example: Ensure that weight loss is not caused claim. If Sonny’s Pharmacy consistently offers shorter wait
by increased exercise by providing evidence or reasoning that times compared to the other two pharmacies, it strengthens
supports the claim that exercise leads to weight loss. the argument that superior customer service is a key factor
6. Validate Data Accuracy: Verify the accuracy and reliability in its consistent profitability.
of the data supporting the causal claim. Any inaccuracies in
Application Drill
the data could undermine the argument, so ensure the data
is sound. Example: Confirm that the studies or information 3. Solve the following question on strengthening of argument
used to establish the link between increased exercise and on your own and to verify your answer, consider the
weight loss are credible and accurate. explanation that follows:
In Simple Terms, We’re like detectives securing our case. Argument: Most managers in the financial industry work
We rule out other suspects, show the cause consistently leads to for several different companies throughout their careers,
the effect, demonstrate the absence of cause means the absence seeking new employment in response to market pressures
of effect, eliminate the chance of a reversed relationship and and changing corporate policies. Paxton Investment
make sure our evidence (data) is spot-on. This approach, though Group, however, is renowned in the financial sector for its
reversed from weakening, is like adding layers of protection to exceptionally low managerial turnover. Paxton attributes
our causal conclusion. By following these steps, we fortify our its ability to retain managers to its extremely generous
argument, making it more robust against doubts or challenges. managerial salaries. Which of the following, if true, would

18 Critical Reasoning PW
provide the strongest support for Paxton Investment Group’s Option (a): This doesn’t directly relate to salary or turnover.
statement regarding its manager’s reluctance to change Family situations might not be a primary driver for Paxton’s
companies? success in retaining managers.
(a) Many managers in the financial sector have spouses who Option (b): This weakens the argument. If managers at
also work in finance. Paxton had a history of working at multiple companies, it
(b) The majority of managers at Paxton Investment Group suggests that turnover might be influenced by factors other
had previously worked at several other companies before than Paxton’s generous salaries.
working at Paxton.
Option (c): This directly supports Paxton’s claim. If income
(c) Managers in the financial industry consistently
is a significant motivator for changing jobs, then Paxton’s
list “income” as the greatest motivator for seeking
generous salaries would indeed contribute to lower turnover.
employment with a new firm.
(d) Investment firms with lower managerial salaries than Option (d): This suggests an alternative compensation
those at Paxton often compensate by offering their strategy but doesn’t strongly support Paxton’s claim about
managers performance-based bonuses. the impact of generous salaries on retention.
Sol. Argument Recap: Paxton Investment Group claims its low Thus, Option (c) provides the strongest support for Paxton’s
managerial turnover is due to its exceptionally generous claim. If managers in the financial industry consistently
managerial salaries. prioritize income, Paxton’s generous salaries would indeed
To strengthen Paxton’s claim, we need an answer choice contribute to the reluctance of its managers to change
that aligns with the idea that generous salaries are indeed a companies.
key factor in low turnover.

Strengthening of an Argument 19
5 Weakening of an Argument

Weaken questions require you to select the answer choice that


“Which of the following, if known, is evidence that contradicts
undermines the author’s argument as decisively as possible.
the hypothesis above?”
Weakening an argument is like finding cracks in a building.
Imagine the argument as a structure built with supporting ideas “Which of the following, if discovered, would be evidence
(premises) leading to a main point (conclusion). Weakening against the speculation above?”
means spotting weak spots or assumptions in this structure. To
weaken, we often go after hidden assumptions—those beliefs that How to Weaken an Argument?
are crucial but not clearly stated. By questioning or disproving Because arguments are made up of premises and conclusions,
these hidden ideas, we weaken the link between the supporting you can safely assume that these are the parts you must attack to
ideas and the main point. It’s like taking out a vital piece from weaken an argument. Let us try to understand the steps to weaken
a puzzle, making the whole picture less convincing. Only by an argument.
understanding the structure of the argument can you gain the  Step 1: Spot the Main Point: Identify the big idea or
perspective necessary to attack the author’s position. conclusion the argument is trying to prove. It’s like finding
Another way is by bringing in information that disagrees the boss in a video game. In an argument, it’s usually the
with the supporting ideas or main point, creating a gap in the most crucial point the author wants you to believe.
logical flow of the argument. Also, arguments often say one  Step 2: Look for Assumptions: Assumptions are like hidden
thing causes another. Weakening can happen by questioning beliefs holding the argument together. Look for things the
or disproving these cause-and-effect relationships. A strong author is taking for granted. For example, if the argument says
weakening move suggests other explanations for what we see. “People love ice cream”, an assumption could be “Everyone
This hints that the main point might not be the only possible can easily afford ice cream”.
explanation. Think of it like exploring different ideas to solve a  Step 3: Question Cause-and-Effect: If the argument says
mystery—each idea weakens the case for one definite answer. one thing causes another, question it. Ask, “Is there another
Additionally, sharing counterarguments or evidence supporting reason this could happen?” For instance, if the argument
a different view weakens the author’s position. It’s like telling claims “Eating chocolate causes happiness”, you might
a different story that challenges the original one’s believability. wonder if other factors contribute to happiness too.
In simple terms, weakening an argument is like gently shaking  Step 4: Introduce other Explanations: Suggest different
the foundation, making the link between supporting ideas and the
ideas that could explain what’s happening. This shows there
main point less sturdy. It’s a skill of finding hidden assumptions,
might be more than one answer. For instance, if the argument
bringing in conflicting facts, and showing other possible views.
says “People buy more umbrellas when it rains”, you might
suggest that people buy umbrellas because they’re on sale,
THE QUESTION STEMS FROM A WEAKENING
not necessarily because it’s raining.
QUESTION
 Step 5: Bring in Counterarguments: Present arguments that
Following are the question stems which help to identify that the go against the main point. It’s like telling a different story
question is asking us to weaken the argument: that challenges the original one. If the argument says “All
“Which of the following, if true, most seriously weakens the students love homework”, you could counter it by saying,
argument above?” “Some students find homework stressful”.
 Step 6: Disprove Supporting Evidence: If the argument
“Which of the following, if true, casts the most doubt on the
relies on facts, question their accuracy. If the facts are shaky,
conclusion drawn above?”
the argument weakens. It’s like checking if the pieces of
“Which of the following, if true, most calls into question the evidence fit together. For example, if the argument states
claim above? “All cars run on solar power”, you might question if that’s
“Which of the following, if true, is most damaging to the true for every single car.
conclusion above?” Weakening is about gently shaking the argument’s foundation.
By finding weak spots, questioning assumptions, and a 16-ounce bottle of water is, on average, over ten times that of the
suggesting other possibilities, you’re like the detective same amount of water run through a home filtration system. Thus,
unravelling the mystery behind the argument. most bottled spring water producers will soon go out of business.
Common Weakening Scenario 1. Which of the following, if true, most severely weakens the
Several scenarios that occur in Weakened question stimuli are argument presented above?
easy to recognize and attack: (a) Several of the companies that bottle and sell spring water
1. Incomplete Information: Sometimes, the author tells only have gone out of business during the past five years.
part of the story. It’s like reading a book missing crucial (b) Because of the inherent costs associated with bottling
chapters. To weaken this, you can introduce new possibilities and transport, most bottled spring water suppliers are
or information that the author overlooked. Imagine finding unable to reduce the wholesale prices they must charge
those missing chapters and showing how they change the tale. for bottled spring water.
2. Improper Comparison: Imagine comparing apples to (c) Most consumers who regularly purchase bottled spring
oranges—they’re different! Similarly, authors might compare water base their beverage purchase decisions exclusively
things that are fundamentally unlike. To weaken this, you on taste preference.
just need to point out why the things being compared aren’t (d) Some people prefer the taste of tap water to that of bottled
alike. It’s like saying, “Wait, these are different!” spring water.
3. Overly Broad Conclusion: Picture someone claiming
Sol. Option (a): This option is incorrect. If several bottled spring
everyone loves pizza when, in reality, only a few people
water companies have gone out of business in recent years,
do. That’s making a conclusion way broader than what the
evidence supports. To weaken this, show why the conclusion it strengthens the author’s argument that most producers will
is too big for the evidence. It’s like adjusting the story to fit soon go out of business.
the facts. Option (b): This option is also incorrect. If bottled water
These are like finding plot holes in a movie—common suppliers cannot reduce wholesale prices due to inherent
problems that, when addressed, make the whole story more costs, it supports the author’s claim that the price difference
will persist, strengthening the argument.
solid. While there are more ways arguments can be weak,
focusing on these three often helps crack the code in CLAT Option (c): This is the correct answer. If consumers base
questions. their bottled water purchases solely on taste preference, it
weakens the argument. The author’s focus is on health and
Incorrect Answer Traps or Common Pitfalls cost, not taste.
Weaken questions on the CLAT can sometimes feel like navigating Option (d): This option is incorrect. The vague term “some”
a maze. Let’s dive into three tricky spots you might encounter: does not provide meaningful information, and the preference
1. Opposite Answers: Imagine intending to head south but for tap water is irrelevant to the argument.
realizing you’re going north—that’s the vibe of opposite Thus option C is the correct answer.
answers. These choices do the exact opposite of what’s
needed. They appear friendly, fitting snugly with the Application Drill
argument, but in reality, they mislead you, strengthening the Read the arguments and solve the question on the weakening of
argument instead of weakening it. an argument. After solving the questions, refer to the explanations
2. Shell Game Answers: Remember the Shell Game from Must to verify your answer.
Be True questions? It’s back. In Weaken questions, it’s like Passage 1
finding puzzle pieces that almost fit but are slightly off. The Argument: Over the past decade, the implementation of artificial
idea from the stimulus shows up, but it’s tweaked enough intelligence (AI) in various industries has led to unprecedented
to be wrong. Watch out for these crafty choices, aiming to levels of efficiency and productivity. Companies that integrate
disrupt conclusions similar to the ones in the stimulus. AI technologies into their operations experience significant cost
3. Out of Scope Answers: Picture driving towards your savings and improved outcomes. Therefore, it can be concluded
destination, and suddenly you’re discussing unrelated that widespread adoption of AI is the key to economic prosperity
detours. Out-of-scope answers do just that—they take you in the future.
off the main road. They bring up issues that don’t connect
to the argument, leaving you wondering why you ventured 2. Which of the following, if true, most severely weakens the
into this irrelevant territory. argument presented above?
(a) Recent studies show that many companies that adopted
Illustration AI technologies faced initial disruptions, resulting in
Passage temporary decreases in productivity and increased costs.
Consumer: The problem with bottled spring water is that it is no more (b) The majority of businesses that have not implemented
healthy than well-filtered tap water, and many companies produce AI technologies report steady growth and profitability
effective water filtration systems for home use. Further, the price of over the past decade.

Weakening of an Argument 21
(c) AI technologies are still in the early stages of 3. Which of the following, if true, most significantly weakens
development, with many potential risks and uncertainties the argument presented above?
regarding their long-term impact on the economy. (a) The construction and maintenance costs of high-speed
(d) Companies that heavily rely on AI technologies often railways are significantly higher than those of traditional
face challenges related to data security and privacy transportation modes, leading to substantial financial
breaches, leading to financial losses and damage to burdens on governments.
reputation. (b) Advances in telecommuting technologies have enabled
Sol. Option (a) This option weakens the argument by presenting many businesses to operate effectively without the need
evidence that challenges the claimed immediate benefits of for frequent physical travel between cities.
AI adoption. If implementing AI can lead to disruptions and (c) Environmental studies indicate that the manufacturing
increased costs, it undermines the notion of widespread cost and disposal of the specialized materials used in high-
savings. speed trains contribute to significant carbon emissions.
Option (b) This option weakens the argument by suggesting (d) Countries with extensive high-speed railway networks
that businesses without AI technologies have still experienced have experienced a decline in tourism revenue, as
growth and profitability, questioning the argument’s premise visitors prefer alternative modes of transportation for
that AI is the sole key to economic prosperity. leisure travel.
Option (c) This option weakens the argument by introducing Sol. Option (a) This option weakens the argument by highlighting
the idea that AI technologies are still in the early stages, the financial challenges associated with high-speed railway
implying that the full impact and consequences are not yet construction. If the costs are substantial, it questions the
known. It introduces an element of uncertainty.
economic feasibility and benefit of such projects.
Option (d) This option weakens the argument by highlighting
Option (b) This option weakens the argument by suggesting
the potential drawbacks of heavy reliance on AI, such
an alternative solution to promoting business activities. If
as security and privacy issues. It introduces the idea that
telecommuting reduces the need for frequent travel, the
the benefits of AI may come with significant risks and
argument’s premise about the economic benefits of high-
downsides.
speed railways is undermined.
The most effective weakening option is (a), as it directly
Option (c) This option weakens the argument by introducing
challenges the immediate positive outcomes of AI adoption
environmental concerns related to the manufacturing and
presented in the argument.
disposal of materials used in high-speed trains. It challenges
Passage 2 the claim of environmental sustainability.
Argument: The construction of a high-speed railway network Option (d) This option weakens the argument by presenting
connecting major cities is essential for economic growth and evidence that extensive high-speed railway networks may
environmental sustainability. High-speed trains are more energy- lead to a decline in tourism revenue. If visitors prefer
efficient than traditional transportation modes, and the reduced alternative modes of transportation, it questions the
travel time between cities promotes business activities and argument’s assumption about the positive impact on tourism.
tourism. Therefore, governments should prioritize investing in Thus. the most effective weakening option is (a), as it
high-speed railways to foster both economic development and addresses both the economic and financial aspects of high-
environmental conservation. speed railway construction.

22 Critical Reasoning PW
6 Understanding Inferences

In our day-to-day conversations, the term “inference” often the lines and making logical connections without the argument
alludes to likely assumptions or probable scenarios. If a friend explicitly telling us what to conclude. For example: “If the
expresses a penchant for chocolate ice cream, we might infer passage talks about rising global temperatures and the decline in
she likes chocolate or perhaps harbours a sweet tooth. However, ice caps, an inference might be that climate change is occurring”.
CLAT takes a more stringent stance. In the CLAT universe, On the other hand, Strengthen questions involve supporting
an inference is not a plausible scenario; it’s an undisputed fact the argument or conclusion provided in the passage. The argument
that unequivocally emanates from the evidence embedded in usually contains a conclusion or a claim, and our task is to find
the argument. Consider the analogy of our friend’s ice cream an answer choice that makes that conclusion more robust or
preference. CLAT insists on certainties: if she declares chocolate believable. It’s like adding evidence or support to make the
as her favourite, she categorically cannot prefer vanilla. The test argument stronger. For example: “If the conclusion is that a new
demands absolutes, a departure from the probabilistic inferences drug is effective in treating a disease, a strengthening answer might
we might draw in everyday situations. provide additional data or studies supporting the drug’s efficacy”.
To navigate the intricacies of CLAT inferences, it’s Thus we can conclude that inference questions ask us to
imperative to internalize their distinct character. Correct uncover implied conclusions based on the information presented,
inferences on the CLAT aren’t mere assumptions; they are without the argument explicitly stating a conclusion. It’s about
ironclad truths derived from the argument’s premises. If someone making logical connections. On the other hand, Strengthen
claims chocolate as a favourite, it’s not a matter of probability; questions require us to find an answer that adds support to the
it inherently excludes the possibility of another flavour being conclusion already present in the argument. One involves drawing
preferred more. For CLAT, inferences are necessitated by the logical inferences, while the other involves making the argument
logic woven into the argument. If a conclusion is what the author more convincing by providing additional support.
explicitly states, an inference is an unspoken truth that logically
Now, Let us understand through some examples of
follows. While conclusions might be debatable or opinion-based,
inferences:
inferences must withstand the scrutiny of unwavering logic.
Example 1: Exclusive Preferences:
DIFFERENCE BETWEEN CONCLUSION AND Argument: Sara claims that hiking is her favourite outdoor
INFERENCE activity.
A conclusion is the author’s explicit statement, laid out for Inference: Sara does not prefer any other outdoor activity over
scrutiny. On the other hand, an inference is an implicit truth, hiking.
seamlessly integrated into the fabric of the argument. The CLAT This inference holds because the term “favourite” implies a
mandates precision, demanding that inferences be undeniably level of exclusivity. If hiking is her favourite, any other outdoor
supported by the argument’s evidence. Consider a scenario activity automatically takes a backseat. If someone claims a specific
where an author concludes that investing in renewable energy preference, like chocolate being their favourite, it necessarily
is economically sound. The conclusion is explicit—the author excludes the possibility of another flavour being preferred more.
asserts that it’s economically viable. An inference, however,
Example 2: Comparative Experiences:
might delve into unspoken certainties, such as the assumption that
renewable energy sources are becoming more cost-effective over Argument: John asserts that the new restaurant in town serves
time, which logically buttresses the economic viability. the best pizza.
Inference: John has tried pizzas from various restaurants to make
DIFFERENCE BETWEEN INFERENCES AND a valid claim.
STRENGTHENING QUESTIONS For John to assert that a particular restaurant serves the best
In inference questions, we are asked to draw conclusions or pizza, he must have sampled pizzas from multiple establishments.
understandings based on the information provided in the argument. The logic hinges on the necessity of comparative experiences
The key here is that the argument itself doesn’t explicitly state a to validate the claim. If the argument discusses a choice or
conclusion; instead, we need to figure out what can reasonably preference, the individual must have had relevant experiences or
be inferred from the information given. It’s like reading between comparisons to make a valid claim.
Inference Question Stems Why are They Tricky?
Following is the non-exhaustive list of types of question stems These traps seem right because they use real-world logic or sound
that will help you to find out that the question wants you to find like they fit. However, the key is to stick to what the argument
out the inference in the passage. directly says. The right answer is something that can be confidently
said based on that info, not just something that kinda makes sense.
Which answer can be “ logically concluded” or the “statements
above most strongly support which of the following Avoiding Traps: Stay focused on what’s in the argument.
conclusions”? Don’t bring in extra info or assume too much. If it’s not spelt out
in the argument, be cautious. The right answer is like a detective’s
Which answer can be “properly inferred”?
conclusion—based on the clues given, not wild guesses.
The statements above “best support” which of the following
“assertions”? Illustration
Argument: The city council has recently implemented a bike-
Which answer “must be true” based upon the above statements?
sharing program, aiming to reduce traffic congestion and promote
“The author implies which of the following extrapolations?” environmentally friendly transportation. The program provides
“What unspoken conclusion is supported by the passage?” convenient bike stations throughout the city where users can rent
bikes for short trips. Based on the information provided, what can
“From the author’s statements, one can infer a conclusion that
be inferred about the city council’s goals?
is not explicitly stated, such as..”.
Following can be the possible inferences:
“The author’s statements logically lead to the conclusion
1. The city council is concerned about traffic congestion and
that..”.
its environmental impact.
“The information in the passage most strongly implies that..”. Inference Explanation: Since the bike-sharing program
“The passage suggests which of the following about..”. targets traffic congestion and promotes environmentally
friendly transportation, it can be inferred that the city council
Common Answer Traps is concerned about these issues.
The most tempting wrong answers on Inference questions tend 2. The city council believes that short-distance trips are a
to revolve around making Real World Inferences—things that we significant contributor to traffic congestion.
would reasonably assume to be true in the real world but don’t Inference Explanation: The implementation of a bike-
have to be true: sharing program for short trips suggests that the city council
1. Going Too Far: This trap makes you think something is true considers short-distance travel as a key factor in traffic
even if it’s not firmly supported by the argument. Imagine congestion.
if someone says they don’t like apples anymore. A tricky
3. The city council values alternative transportation methods
answer might say they now love oranges. It sounds logical,
to reduce reliance on private cars.
but the original statement only talked about a change, not the
complete opposite. Inference Explanation: The introduction of a bike-sharing
2. Too Specific or Too General: Sometimes, wrong answers program indicates a commitment to alternative transportation,
give you a specific detail or a broad idea that’s not entirely emphasizing a desire to reduce the dependence on private cars.
backed up by the argument. If the argument says it’s raining Application Drill
today, a wrong answer might say there’s a hurricane. The Read the following passages and solve the questions on inference.
first statement only talked about rain, not a big storm.
To verify your answer, consult the explanations that follow:
3. Reverse Direction or Switch Terms: This tricky move
involves changing the order of things or switching words 1. Argument: A recent study found a correlation between
around. If someone says eating cake causes happiness, a regular exercise and improved cognitive function in adults
wrong answer might say happiness causes people to eat cake. over the age of 50. The participants who engaged in physical
It seems close, but it’s flipping the cause and effect. activity three times a week showed a significant increase
4. Out of Scope: These answers bring in new info that wasn’t in in memory retention and problem-solving skills compared
the argument. Imagine someone says they’re reading a book. to those who did not exercise regularly. Based on the
A wrong answer might say they love watching movies. It’s information provided, what can be reasonably inferred about
not about what was said; it’s adding unrelated things. the relationship between exercise and cognitive function in
5. Ignoring Possibilities: Sometimes, wrong answers ignore adults over 50?
other possible scenarios. If someone says they’re not eating (a) Adults who exercise regularly will never experience
cookies anymore, a tricky choice might say they’ve switched cognitive decline.
to ice cream. It sounds reasonable, but maybe they’re not (b) Engaging in physical activity at least three times a week
having desserts at all! is a guarantee of enhanced cognitive function.

24 Critical Reasoning PW
(c) The study suggests a positive association between regular (a) Work-life balance, workload, and recognition are the
exercise and improved memory and problem-solving only factors contributing to job satisfaction at XYZ
skills in adults over 50. Corporation.
(d) All adults over 50 will experience the same degree of (b) Other external factors, not mentioned in the survey, play
cognitive improvement if they exercise regularly. a negligible role in influencing job satisfaction among
XYZ Corporation employees.
Sol. Option (a) This statement is too extreme. While the
study shows a correlation, it doesn’t mean that exercising (c) Addressing issues related to work-life balance, workload,
guarantees complete immunity from cognitive decline. It and recognition could potentially lead to an improvement
overstates the findings and goes beyond what the study in job satisfaction among XYZ Corporation employees.
suggests. (d) Employees at XYZ Corporation have never been
Option (b) Similar to option (a), this statement is too satisfied with their jobs.
absolute. The study demonstrates a correlation, but it Sol. Option (a) This option is too extreme. While the mentioned
doesn’t provide a guarantee that everyone who exercises factors are highlighted in the survey, it doesn’t mean that
will experience enhanced cognitive function. they are the sole contributors to job satisfaction. There could
Option (c) Correct. This option accurately reflects the be other factors not explicitly mentioned.
inference based on the study. The study establishes a positive Option (b) This statement is too restrictive. The survey
association between regular exercise and improved memory only focused on certain factors, and it doesn’t imply that
and problem-solving skills in adults over 50. external factors have a negligible role. It’s an unwarranted
Option (d) This option is too broad. The study doesn’t assumption.
make a claim about every individual experiencing the same Option (c) Correct. This option aligns with a reasonable
degree of cognitive improvement. It focuses on a correlation, inference. The survey identifies specific issues affecting
acknowledging variations among participants. job satisfaction, and addressing these concerns could lead
Thus, Option (c) appropriately captures the essence of to an improvement. It doesn’t make absolute claims about
the study’s findings without going beyond the established the guaranteed outcome.
correlation. Option (d) This option is an extreme statement and goes
2. Argument: A recent survey conducted among employees beyond the scope of the survey. The survey reflects a
of XYZ Corporation revealed a significant decrease in job snapshot in time and doesn’t imply perpetual dissatisfaction
satisfaction levels. The employees cited inadequate work- among employees.
life balance, increased workload, and minimal recognition Thus, Option (c) appropriately extends from the survey
for their contributions as the primary reasons for their findings without making sweeping statements about the
dissatisfaction. What can be reasonably inferred about the exclusivity of factors or the perpetual dissatisfaction of
factors influencing job satisfaction among employees at employees.
XYZ Corporation?

Understanding Inferences 25
7 Cause and Effect Reasoning

WHAT IS THE MEANING OF CAUSALITY? Was a factor of


When we try to understand events, we often look for reasons
Is an effect of
behind why things happened. This natural inclination leads us to
cause-and-effect reasoning, where we claim that one thing causes Causality in Premise and Causality in Conclusions
another or is caused by another. On the CLAT, you’ll encounter Understanding how cause-effect statements function within
cause-and-effect reasoning in many Critical Reasoning problems, arguments is crucial for navigating through reasoning problems.
typically in conclusions where an author mistakenly asserts that Causal statements in an argument can either be in the premise or
one event is the direct cause of another. For instance: “Last week, the conclusion. The distinction between the two holds the key to
Apple announced a quarterly deficit, and the stock market dropped identifying potential flaws in the reasoning. If the causal assertion
10 points. Thus, Apple’s announcement must have caused the appears in the conclusion, or if the conclusion presumes a cause-
drop”. and-effect relationship, there’s a higher likelihood of flawed
However, most causal conclusions on the CLAT are flawed reasoning. Let’s delve into examples to illustrate these scenarios.
because there could be alternative explanations for the claimed Example 1: Causal Conclusion:
relationship. Another cause might explain the effect, a third event
Premise: In North America, people drink a lot of milk.
might have caused both the stated cause and effect, the situation
might be reversed, or the events might be related but not in a Premise: There is a high frequency of cancer in North America.
causal way. Sometimes, the entire occurrence could be the result Conclusion: Therefore, drinking milk causes cancer.
of chance. In simple terms, causality means saying that one event In this instance, the author concludes that drinking milk
makes another event happen. The cause is the event that triggers causes cancer-based on the observed correlation. However, this
the other event, and the effect is the event that follows the cause. conclusion is flawed, as correlation does not necessarily imply
It’s crucial to note that the cause must happen before the effect, causation. This example reflects a common type of error—
and the cause is like the activator or ignitor in this relationship. attributing causation without sufficient evidence.
The effect always occurs after the cause in terms of time.
Example 2: Causal Premise
Signifiers of Cause and Effect Premise: Drinking milk causes cancer.
Following is the list of words which help us to introduce cause Premise: The residents of North America drink a lot of milk.
and effect relationship:
Conclusion: Therefore, in North America, there is a high
Caused by frequency of cancer among the residents.
Responsible for Here, the author begins with a causal principle—establishing
that drinking milk causes cancer—and logically extends it to a
Because of conclusion. This form of reasoning is generally considered valid.
Reason for While the CLAT often presents problems in a format similar to the
first example, where causal conclusions are flawed, it occasionally
Leads to
introduces problems akin to the second example. Crucially,
Induced by if a causal claim is made in the premises, it doesn’t inherently
Promoted by introduce a causal reasoning error. It tends to permit premises to
go unchallenged, focusing more on the reasoning that follows.
Determined by Authors are generally allowed to state a causal relationship in the
Produced by premise and build their argument from there.
Product of Understanding these nuances helps test-takers distinguish
between valid causal reasoning and flawed causal reasoning. It
Played the role of emphasizes the importance of critically examining where the
causal assertion is situated within an argument. This skill is
pivotal for addressing questions that involve cause-and-effect Consider an example to illustrate this CLAT-centric
relationships, enabling test-takers to identify and navigate through assumption:
reasoning errors effectively. Premise: Average temperatures are higher at the equator than in
The Situation of Errors in Causality any other area.
Causal conclusions often emerge in Critical Reasoning questions Premise: Individuals living at or near the equator tend to have
based on two common scenarios, each rooted in the temporal lower per-capita incomes than individuals living elsewhere.
relationship between events. Conclusion: Therefore, higher average temperatures cause lower
Scenario 1: Temporal Sequence (One Event Precedes Another) per-capita incomes.
When one event happens before another, there’s a common but This argument typifies flawed causal reasoning. While
misleading tendency to assume a cause-and-effect relationship. the premises establish a connection (living at the equator), the
The classic example of a rooster crowing before the sun rises conclusion erroneously asserts that one element directly causes
illustrates the potential flaw in such reasoning. Despite the the other. Speakers believe this singular cause-and-effect
relationship, considering it exclusive. In the real world, one might
rooster’s consistent crowing preceding sunrise, it is erroneous to
entertain alternative causes, such as a lack of natural resources
conclude that the rooster causes the sun to rise. This whimsical
affecting per-capita income, but CLAT reasoning dismisses these
example highlights the danger of hastily attributing causation
possibilities. The powerful assumption comes to the forefront
based solely on temporal sequence.
in Weaken, Strengthen, and Assumption questions. In Weaken
Scenario 2: Simultaneous Occurrence of Events questions, test-takers aim to challenge the assumed exclusivity of
Another scenario leading to causal conclusions involves events the cause-effect relationship. By introducing alternative causes or
occurring simultaneously. Here, the assumption that one event demonstrating that the stated cause does not always lead to the
causes the other is again challenged. While it’s plausible that one effect, one weakens the argument.
event may cause another, it’s equally possible that both events are Example:
influenced by a third factor or are merely correlated without direct
Premise: A study concludes that increased coffee consumption
causation. leads to higher productivity at work.
A case in point is the correlation between ice cream Conclusion: Therefore, employers should encourage their
consumption and the murder rate. Concluding that consuming employees to drink more coffee for better performance.
ice cream causes a higher likelihood of committing murder
oversimplifies the relationship. In reality, both phenomena may To weaken this, one might introduce alternative causes
be outcomes of a shared cause—in this case, hot weather. When like improved office environment or efficient time management
leading to increased productivity, challenging the assumption that
temperatures rise, people are more inclined to consume ice cream,
coffee consumption is the sole cause.
and there may also be an increase in crime rates. This example
serves to caution against accepting causal connections without Effect on Strengthen and Assumption Questions:
considering alternative explanations. Understanding these Understanding this assumption is equally critical in Strengthen
scenarios is vital for critical reasoning, as the CLAT often presents and Assumption questions. In Strengthen questions, correct
arguments with temporal relationships between events. The answers often affirm the exclusivity assumption, reinforcing the
pitfalls lie in assuming causation based solely on chronological idea that the stated cause is the only contributor to the effect.
order or simultaneous occurrence. Test-takers need to exercise In Assumption questions, correct assumptions align with this
caution, recognizing that correlation does not always imply powerful belief, reinforcing the singular causal connection.
causation and considering alternative explanations for observed
How to Attack a Causal Conclusion?
patterns.
Weakening a cause-and-effect relationship in Weaken questions
The Central Assumption of Causal Conclusions typically involves executing one of the following tasks:
Understanding the underlying assumption in causal conclusions A. Find an alternate cause for the stated effect: Since the
is crucial for navigating Weaken, Strengthen, and Assumption author assumes there is only one cause, discovering another
questions. A common misconception is assuming that CLAT’s potential cause undermines the conclusion. This strategy
causal reasoning aligns with real-world assumptions. In reality, challenges the exclusivity of the stated cause and effect.
CLAT speakers make extreme assumptions, believing that the Example: If an argument claims that increased coffee
stated cause is the sole factor responsible for the effect, dismissing consumption boosts workplace productivity, proposing
all other potential causes. In real-world causal thinking, people alternative causes like a collaborative office environment or
recognize that various causes can lead to the same effect, efficient task management weakens the notion that coffee is
acknowledging a multitude of cause-and-effect scenarios. the sole catalyst.
However, CLAT diverges from this by assuming exclusivity: if B. Show that even when the cause occurs, the effect does not
one occurrence is deemed the cause, the CLAT speaker asserts occur: This approach often manifests as a counterexample.
that it is the only cause and that it unfailingly produces the effect. Given the author’s belief that the cause invariably produces

Cause and Effect Reasoning 27


the effect, any scenario where the cause happens, yet the doesn’t cause drowning; it’s just hot outside, so people swim
effect does not, weakens the conclusion. Example: If a more. Now, causation is a more serious commitment. It’s saying,
statement asserts that regular exercise guarantees weight “Hey, this thing is making that thing happen”. Picture a rooster
loss, presenting instances where individuals exercise but do crowing before sunrise. Does it make the sunrise? Of course not!
not lose weight challenges the assumed direct link between The crowing and sunrise correlate, but the rooster doesn’t cause
exercise and weight loss. the sun to rise. In CLAT terms, causation insists that one event
C. Show that although the effect occurs, the cause did not directly triggers another, like flipping a switch turns on a light.
occur: Similar to the previous tactic, this can take the form So, be cautious when folks mix up correlation and causation.
of a counterexample. As the author assumes a consistent In the 1940s, some scientists thought antibiotics were superheroes,
relationship between cause and effect, any situation where the making sick folks stronger. But wait—taking antibiotics and
effect materializes without the cause weakens the conclusion. getting better could simply correlate; it doesn’t mean the medicine
Example: If an argument contends that studying late at caused the improvement. Maybe folks got better because they
night guarantees academic success, demonstrating cases rested more or ate chicken soup. Let’s dive deeper. Imagine a
where students succeed without late-night study sessions graph. If as ice cream sales soar, so do the number of sunburn
undermines the claim of a necessary causal connection. cases, we see a correlation. But claiming that ice cream causes
D. Show that the stated relationship is reversed: Given sunburn is silly. They’re related because both rise with heat,
the author’s belief in the accuracy of the cause-and-effect not because one causes the other. CLAT loves to throw these
relationship, revealing that the relationship is inverted curveballs, where things seem connected, but you need to ask,
undermines the conclusion. This tactic challenges the “Is it a real cause, or are they just hanging out together?” Now,
correctness of the claimed cause and effect. Example: If an let’s spice it up with reverse causation. Think about umbrellas and
argument suggests that increased ice cream consumption rain. If we observe people carrying umbrellas and assume they’re
causes warmer weather, demonstrating that warmer weather causing the rain, that’s like saying the effect (rain) causes the
is the cause of increased ice cream consumption contradicts cause (umbrellas). But, come on, umbrellas can’t summon rain! In
the stated relationship. the CLAT world, watch out for arguments where the effect seems
E. Show that a statistical problem exists with the data used to cause the cause—it’s a logic no-no. You might also encounter
to make the causal statement: When the data supporting a sneaky scenarios where a third amigo influences both events,
causal statement are flawed, the validity of the entire causal leading to a correlation. For instance, the more firefighters at a
claim is questionable. Attacking the statistical foundation scene, the more damage. It’s not that firefighters cause destruction,
calls into question the reliability of the cause-and-effect but big fires attract more firefighters. So, be on guard for situations
assertion. Example: If an argument relies on data suggesting where a hidden player affects both things you’re looking at.
a direct link between a specific diet and reduced stress, CLAT wants you to be a detective, not a blind follower.
exposing flaws or biases in the data weakens the confidence If you see two events together, don’t jump to conclusions. Ask
in the causal claim. questions: Is there a direct cause, or are they just good pals
In summary, tackling a causal conclusion in Weaken questions hanging out? Remember, correlation isn’t causation. People
involves challenging the assumed singular cause-and-effect getting sunburned and eating ice cream doesn’t mean the cone
relationship. Whether by proposing alternative causes, providing causes the burn. Ice cream and sunburn may just love summer
counterexamples, exposing instances where the cause does not lead to equally. In a nutshell, correlation is like friends showing up at
the effect, revealing cases where the effect occurs without the cause, your happy party, and causation is one friend throwing a confetti
demonstrating a reversed relationship, or questioning the statistical bomb, making everyone cheer. Keep your logic radar sharp, and
integrity of the data, these strategies aim to erode the foundation of don’t be fooled by things happening together. The key? Always
the causal claim and introduce doubt into the argument. ask, “Is it a real cause, or are they just along for the ride?”

Correlation v. Causation Illustration


Understanding the difference between correlation and causation 1. Which of the following statements best represents a cause-
is crucial, and it’s like distinguishing between having an umbrella and-effect relationship? Choose the option where one
and it raining. Just because you see people with umbrellas doesn’t statement can be reasonably considered the cause, and the
mean they caused the rain, right? Similarly, when two things happen other the effect.
together, it’s called correlation. But don’t rush to say one thing (a) The city experienced a surge in tourism after implementing
causes the other—that’s causation, and it’s trickier. Correlation a comprehensive marketing campaign.
is like having a buddy who always shows up when you’re happy. (b) The student’s academic performance improved
Your friend’s presence correlates with your happiness, but it significantly following the introduction of a new
doesn’t mean your friend causes your joy. Likewise, when things teaching method.
correlate, they happen at the same time, but there’s no guarantee (c) The company decided to invest in renewable energy
one causes the other. For instance, ice cream sales and drowning sources, leading to a reduction in its overall carbon
incidents both rise in summer. Correlation, right? But ice cream footprint.

28 Critical Reasoning PW
(d) As temperatures rose, the demand for ice cream increased individual differences in responsiveness but doesn’t directly
in the local ice cream parlour. challenge the predicted overall impact of antibiotics.
Sol. Option (a) In this option, the implementation of a Option (d) This statement directly addresses a reasoning
comprehensive marketing campaign is the cause, leading error in the scientists’ prediction. It points out that strong
to the effect of a surge in tourism. The marketing campaign antibiotics may not distinguish between harmful and
beneficial bacteria, posing a potential threat to the patient’s
is the action that precedes and influences the increase in
health. This aligns with the prediction error of assuming
tourism.
only harmful bacteria would be targeted.
Option (b) The introduction of a new teaching method is
Option (d) directly identifies a flaw in the scientists’
the cause, and the improved academic performance is the
prediction by highlighting that strong antibiotics may
effect. The change in the teaching method precedes and
affect both harmful and beneficial bacteria, contrary to
contributes to the subsequent improvement in students’
the assumption that patients would only benefit from the
academic performance.
destruction of harmful bacteria. In summary, option (d) is
Option (c) Investing in renewable energy sources is the the correct answer as it specifically addresses the reasoning
cause, resulting in the effect of a reduction in the company’s error in the scientists’ prediction, while the other options
overall carbon footprint. The decision to invest precedes and either provide additional context or discuss factors that are
causes the observed decrease in carbon footprint.
not central to the prediction flaw.
Option (d) The rise in temperatures is the cause, leading to
the effect of increased demand for ice cream. The change in Application Drill
temperature precedes and influences the subsequent increase Read the following arguments and solve the questions that follow.
in ice cream consumption. Refer to the explanations for any clarification.
In summary, each option in this question illustrates a cause- Argument 1: Alpha Cola, the best-selling soft drink nationally
and-effect relationship, with one statement representing the among soda drinkers aged 18 to 25, recently completed an
cause and the other the effect. Understanding the sequence expensive and successful ad campaign. The makers of Epsilon
of events and the influencing factor helps identify the causal Cola, a less popular soft drink that has been on the market for
relationship in each scenario. many years, claim that without the recent ad campaign, Alpha
2. Many scientists of the 1940s predicted that new, exceptionally Cola would be no more popular than Epsilon.
potent antibiotics would soon revolutionize the entire 3. Which of the following, if true, would cast the most serious
medical field. Patients would be given large dosages of these doubt on the assertion of the makers of Epsilon Cola?
antibiotics, which would attack and kill harmful bacteria in (a) Alpha Cola’s recent ad campaign was intended in part
the body, making the patients stronger as a result. Which of to increase sales of the soft drink to soda drinkers aged
the following, if true, best describes a reasoning error in the 18 to 25.
scientists’ prediction? (b) Beverage buying decisions can be significantly
(a) To achieve the proper dosage requirements, several influenced by effective ad campaigns.
rounds of antibiotics would likely be necessary. (c) Alpha Cola’s recent advertising campaign was one of
(b) In the 1940s, antibiotics had only recently been the most expensive advertising campaigns in history.
discovered. (d) Before the recent campaign, Alpha Cola had never
(c) Some patients respond more quickly than others to strong advertised but had significantly outsold all other soft
antibiotics. drinks on the market for several years.
(d) Strong antibiotics act on all bacteria in the body in the Sol. Option (a) This statement supports the idea that the recent
same manner, including beneficial bacteria critical to ad campaign of Alpha Cola targeted the same age group
human health. as its primary consumer base. However, it doesn’t directly
Sol. Option (a) This statement addresses the administration of challenge the assertion made by the makers of Epsilon
antibiotics but doesn’t directly challenge the scientists’ Cola. It is consistent with Alpha Cola’s efforts to maintain
prediction. It doesn’t pinpoint a reasoning error in the or enhance its popularity among the specified age group.
prediction; instead, it discusses the practicality of achieving Option (b) This statement provides a general observation
the proper dosage. about the influence of ad campaigns on buying decisions.
Option (b) This statement provides historical context While it establishes the effectiveness of ad campaigns in
but doesn’t inherently point out a reasoning error in the influencing consumer choices, it doesn’t specifically address
scientists’ prediction. It introduces a fact about the timeline whether Alpha Cola’s recent success is solely due to its ad
of antibiotic discovery but doesn’t directly address the campaign.
scientists’ expectations. Option (c) This statement emphasizes the cost of Alpha
Option (c) While this statement introduces variability Cola’s recent ad campaign but does not directly challenge
in patient responses, it doesn’t specifically highlight a the assertion made by the makers of Epsilon Cola. The
reasoning error in the scientists’ prediction. It acknowledges expense of the campaign does not necessarily undermine

Cause and Effect Reasoning 29


the claim that Alpha Cola would be no more popular than Sol. Option (a) This information doesn’t directly undermine the
Epsilon without it. argument. If the same scale is used consistently, it ensures
Option (d) This statement introduces crucial information consistency in measurements. This option doesn’t challenge
about Alpha Cola’s performance before the recent ad the reasoning behind advising the patient to gain more
campaign. Specifically, it highlights that Alpha Cola had weight.
never engaged in advertising before the recent campaign, Option (b) This option doesn’t necessarily undermine the
yet it had been highly successful and had outsold all other argument. Whether or not the patient was informed about
soft drinks on the market for several years. the weight gain doesn’t address the reasoning behind the
In essence, option (d) weakens the claim made by the makers advice to gain more weight.
of Epsilon Cola by providing an alternative explanation for Option (b) This option has the potential to weaken the
Alpha Cola’s popularity, indicating that factors other than argument. If the patient has eaten less than usual and still
the recent ad campaign contributed to its success. gained weight, it suggests that factors other than increased
Argument 2: Medical Student: Last week, a certain patient at this food intake may be contributing to the weight gain,
hospital weighed 150 lbs. Since the same patient weighs 160 lbs. undermining the student’s advice.
today, and he appears to be much healthier than he was last week, Option (d) This is the correct answer. It directly challenges
he would be well advised to gain another ten pounds during the the student’s argument. If the patient’s weight last week
coming week. was influenced by an illness, the current weight gain may
4. Which of the following, if true, undermines the argument be a rebound from the previous illness rather than a signal
of overall health improvement.
above?
The medical student’s advice is based on the assumption
(a) The same scale was used to measure the patient’s weight
that the patient’s weight gain is a positive sign of improved
in both instances.
health. However, if the patient’s weight drop last week
(b) The patient was notified by his physician of this week’s was due to illness, gaining back that weight doesn’t
weight gain. necessarily indicate a healthy state. It might be a recovery
(c) During the past week, the patient has eaten less food from the illness rather than a need for additional weight
than he would normally eat. gain. In conclusion, option (d) undermines the argument
(d) When the patient was weighed last week, an illness had by suggesting an alternative explanation for the observed
caused the patient’s weight to drop ten pounds below its weight change, indicating that the weight gain might not be
normal level. a straightforward indicator of improved health.

30 Critical Reasoning PW
8 Resolving Paradox
in an Argument

In the realm of Critical Reasoning questions, resolving paradoxes 3. Semantic Paradoxes: Arising from language and meaning,
becomes a crucial skill, and understanding what a paradox is these paradoxes delve into the complexities of self-referential
can significantly enhance one’s ability to navigate complex statements and linguistic structures. Semantic paradoxes
arguments. A paradox in this context refers to a situation where revolve around language and meaning. The “Epimenides
two seemingly contradictory or conflicting pieces of information paradox” is an instance where a statement refers to itself.
coexist, creating a puzzle that demands resolution. The importance Epimenides, a Cretan, stated that all Cretans are liars. If his
of unravelling paradoxes lies in the fact that they expose logical statement is true, it implies that he, being a Cretan, is also
gaps or inconsistencies in an argument, inviting us to dig deeper a liar, leading to a paradox. This type of paradox highlights
into the underlying premises and conclusions. Identifying and the complexities embedded in self-referential statements.
resolving paradoxes is akin to untangling knots in the fabric 4. Set-Theoretic Paradoxes: Within the realm of mathematics,
of reasoning, revealing insights that may not be immediately paradoxes related to set theory, like Russell’s paradox, have
apparent. To illustrate, consider a scenario where a company’s played a crucial role in shaping mathematical foundations.
profits are soaring, but customer satisfaction is purportedly Set-theoretic paradoxes emerge within mathematical
plummeting. This paradox prompts us to question the relationship frameworks. The “Russell’s paradox” is a well-known
between financial success and customer contentment. Spotting example. Consider a set of all sets that do not contain
paradoxes involves keen observation and an ability to discern themselves. If such a set exists, it leads to a contradiction
discrepancies within the narrative. when determining whether it contains itself or not. Russell’s
paradox played a crucial role in the development of set theory
TYPES OF PARADOXES in mathematics.
Paradoxes are intriguing phenomena that challenge our 5. Time Travel Paradoxes: Explored in physics and philosophy,
understanding of logic and reasoning. They often involve situations these paradoxes arise when considering the potential logical
where seemingly contradictory or counterintuitive elements inconsistencies in time travel scenarios. Time travel
coexist, creating a puzzle that requires careful examination to paradoxes involve situations where time travellers face
resolve. Let’s delve into different types of paradoxes, exploring logical inconsistencies. The “grandfather paradox” is a classic
their characteristics and providing examples to illustrate each example. If someone travels back in time and prevents their
category. grandfather from meeting their grandmother, it raises the
1. Logical Paradoxes: Common in discussions about self- question of the time traveller’s existence. Resolving this
reference and logical contradictions, they challenge the paradox requires considering alternative timelines or parallel
coherence of certain statements. Logical paradoxes arise from universes.
self-reference or logical contradictions within a statement. 6. Sorites Paradox: Examining the challenges in defining
One classic example is the famous “liar paradox”. Imagine boundaries and gradual changes, this paradox often emerges
someone saying, “I always lie”. If the statement is true, then in discussions about vague concepts. The sorites paradox,
the person is indeed lying, leading to a logical contradiction. also known as the paradox of the heap, challenges our
On the other hand, if the statement is false, then the person understanding of boundaries and gradual changes. If we start
does not always lie, again resulting in a contradiction. with a heap of sand and gradually remove individual grains, at
2. Observational Paradoxes: Often encountered in what point does it cease to be a heap? The paradox highlights
scientific contexts, where empirical evidence may defy the difficulty in defining precise boundaries in concepts like
initial expectations, leading to apparent contradictions. “heap” or “bald”.
Observational paradoxes involve situations where empirical 7. Ship of Theseus Paradox: A thought experiment exploring
evidence or observations defy our expectations. An example identity and continuity, particularly relevant in philosophical
is the “twin paradox” in the theory of relativity. According discussions about the persistence of identity through change.
to Einstein’s theory, if one twin travels at near-light speed The Ship of Theseus paradox explores identity and continuity.
into space and returns, they would be younger than their If every part of a ship is replaced over time, is it still the
Earth-bound sibling due to time dilation. This outcome, same ship? This thought experiment raises questions about
counterintuitive from a common-sense perspective, is the essence of identity and how much change an object can
supported by experimental evidence. undergo while retaining its original identity.
8. Zeno’s Paradoxes: Zeno’s paradoxes, originating in ancient Resolving paradox questions in Critical Reasoning on the CLAT
Greek philosophy, challenge our understanding of motion and involves navigating through seemingly contradictory information
infinity. One of these paradoxes involves Achilles racing a and finding a logical explanation to reconcile the apparent conflict.
tortoise. According to Zeno, Achilles, despite being faster, Let’s break down the process into simple steps:
can never overtake the tortoise because he must first reach  Step 1: Understand the Paradox: The first and crucial step
the point where the tortoise started, then the halfway point, is to fully comprehend the paradox presented in the argument.
and so on, leading to an infinite series of tasks. Identify the contrasting elements or pieces of information
9. Barber Paradox: An example of a logical paradox involving that seem to be at odds with each other. This could involve
self-reference, questioning the logical coherence of a specific unexpected outcomes, seemingly contradictory facts, or
scenario. The barber paradox illustrates self-reference in a situations that defy common expectations.
logical context. Consider a town where the barber shaves  Step 2: Identify the Core Elements: Pinpoint the
all those who do not shave themselves and only those. central elements of the paradox. What are the key pieces
The question arises: Who shaves the barber? If he shaves of information or conditions that create the apparent
himself, it contradicts the rule that he shaves only those who contradiction? Clearly understanding the core of the paradox
do not shave themselves, and if he doesn’t shave himself, it will guide your search for a resolution.
contradicts the rule that he shaves all those who do not shave  Step 3: Explore Answer Choices: Carefully read through
themselves. the answer choices. Keep in mind that the correct answer will
10. Tristram Shandy Paradox: Emerging from literature, this provide new information or considerations that, when taken
paradox explores the challenges of representing a complete into account, help reconcile the conflicting elements. As you
life narrative and the interplay between time and storytelling. evaluate each choice, ask yourself how it impacts the paradox
The Tristram Shandy paradox arises from the fictional work and whether it offers a plausible explanation.
“The Life and Opinions of Tristram Shandy, Gentleman” by  Step 4: Consider Both Sides: Remember that a good
Laurence Sterne. In the book, Tristram attempts to chronicle resolution should address both sides of the paradox. It should
his life but becomes so absorbed in details that the narrative not dismiss or ignore any of the conflicting information
never catches up with the present. This paradox questions but rather provide a coherent connection between them.
the feasibility of accurately representing one’s life through Be cautious of choices that seem to favour one side while
a comprehensive narrative. neglecting the other.
Question Stems of Paradox  Step 5: Look for New Information: The correct answer is
Following is a non-exhaustive list of questions that will help you likely to introduce new information that was not explicitly
to identify a question on a paradox: mentioned in the original argument. This new information
acts as the missing link that ties together the seemingly
“Which of the following, if true, most helps to resolve the contradictory elements. Be attentive to details and consider
apparent paradox?” how each choice adds to your understanding of the situation.
“Which of the following, if true, does the most to explain the  Step 6: Eliminate Irrelevant Choices: Discard answer
result described above?” choices that do not contribute to resolving the paradox or,
“Which of the following, if true, best accounts for the seeming worse, exacerbate the contradiction. If an option introduces
discrepancy described above?” irrelevant information or fails to address the core elements
of the paradox, it is likely incorrect.
“Which of the following hypotheses best explains the contrast
described above?”  Step 7: Anticipate the Outcome: Before selecting your
final answer, think about how the introduced information or
“Which of the following statements, if accurate, helps make explanation will impact the overall scenario. Visualize how
sense of the apparent contradiction outlined in the argument?” the resolution would alter the initial paradox and lead to a
“Which of the following, if true, harmonizes the conflicting more coherent understanding.
information presented?”  Step 8: Verify Against the Paradox: Once you have a
“Which of the following, if true, provides the most reasonable potential answer, double-check that it indeed addresses the
explanation for the observed inconsistency?” paradox. Ensure that the new information complements both
sides of the conflict, creating a more cohesive and logical
“Which of the following, if true, resolves the puzzle raised by
framework.
the coexistence of these seemingly contradictory facts?”
 Step 9: Prioritize Simplicity: Choose the most straightforward
Steps to Solve a Paradox Question resolution. While it’s tempting to overcomplicate the solution,
Following is the suggestive way to resolve a paradox in an the correct answer often provides a clear and concise
argument: explanation without unnecessary complexities.

32 Critical Reasoning PW
Common Pitfalls and Traps in Paradox Questions by introducing unnecessary complexities can make the
Paradox questions can be tricky, like solving a puzzle where puzzle harder than it needs to be. Look for straightforward
things seem a bit mixed up. But don’t worry, we’ll explore some explanations before diving into intricate possibilities.
common traps or pitfalls you might encounter when dealing with In summary, approaching paradox questions requires a
paradoxes. Let’s dive in and make sense of these challenges in careful and patient mindset. Keep an eye out for assumptions,
simple terms: consider the context, and don’t be swayed by distractions. Stay
1. Overlooking Assumptions: Sometimes, we make open-minded, explore alternatives, and pay attention to details.
assumptions without even realizing it. In paradox questions, By avoiding these common pitfalls, you’ll be better equipped to
there might be hidden assumptions that, if overlooked, can unravel the mysteries hidden within paradoxes.
lead to confusion. Always be on the lookout for these sneaky Illustration
assumptions that could be causing the apparent contradiction.
1. Paradox: In a small town, the average income of residents
2. Jumping to Conclusions: It’s like trying to finish a story
is significantly lower than the national average. However,
without reading all the chapters. If you rush to a conclusion
the residents report higher levels of satisfaction with their
too quickly, you might miss crucial details. Take your time
financial well-being compared to the national average. How
to understand the entire scenario before deciding what makes
can a town with lower incomes boast higher satisfaction?
sense and what doesn’t.
(a) The small town has a lower cost of living than the
3. Ignoring Context: Imagine trying to understand a joke
national average.
without knowing the context—it might not make much sense.
Similarly, in paradox questions, the context is vital. Ignoring (b) Residents in the town receive financial assistance
or misunderstanding the context can lead you astray. Pay programs.
attention to the whole story, not just bits and pieces. (c) T h e n a t i o n a l a v e r a g e i n c l u d e s h i g h - i n c o m e
4. Getting Distracted: Picture trying to solve a puzzle with neighbourhoods that skew the overall figure.
distractions all around. In paradox questions, there might (d) The residents in the town prioritize non-material aspects
be tempting but irrelevant information that can lead you off of life over income.
course. Stay focused on what truly matters to unravel the Sol. Analysis: This paradox presents a contradiction between
paradox. the reported satisfaction levels and the average income.
5. Assuming Extreme Cases: Sometimes, we think in Residents seem content despite earning less on average.
extremes, like imagining the fastest car or the tallest building. To resolve this paradox, we need to find information
In paradox questions, assuming extreme cases without that explains why lower-income residents might still feel
considering the middle ground can trip you up. Look for a financially satisfied.
balanced understanding rather than jumping to extremes. Option (a): This provides a potential explanation. If the cost
6. Not Considering Alternatives: It’s like having only one of living is lower, residents can maintain satisfaction despite
flavour of ice cream when there are so many others to try. If lower incomes. Keep it for now.
you stick to a single idea without exploring alternatives, you Option (b): Financial assistance could contribute to
might miss the solution to the paradox. Keep an open mind satisfaction but doesn’t directly address the paradox. Set
and consider various possibilities. aside.
7. Forgetting Nuances: Details matter, just like the ingredients Option (c): Explains why the national average is high but
in a recipe. Forgetting or overlooking small details in a doesn’t clarify the town’s satisfaction paradox. Eliminate.
paradox can lead to confusion. Pay attention to nuances,
Option (d): Offers an alternative perspective, suggesting that
as they often hold the key to resolving the apparent
satisfaction is not solely tied to income. Keep it for now.
contradiction.
Now, compare the remaining options:
8. Misinterpreting Language: Language can be like a puzzle
itself, with words having different meanings. Misinterpreting Option (a): The lower cost of living is a plausible explanation
the language used in a paradox can throw you off track. Take for residents’ satisfaction.
time to understand the meaning of words and phrases to avoid Option (d) Prioritizing non-material aspects aligns with the
falling into this trap. paradox but doesn’t directly address the lower income.
9. Assuming Personal Beliefs: Imagine trying to solve a Option (a) provides a more direct and straightforward
mystery while sticking to your ideas. If you let personal resolution to the paradox by explaining how a lower cost of
beliefs cloud your judgment, it might be challenging to see living contributes to residents’ higher satisfaction. Therefore,
the paradox objectively. Stay open-minded and approach the (a) is the most suitable answer. This example demonstrates
question without letting your personal opinions take over. the process of identifying the central elements of a paradox,
10. Overcomplicating Solutions: Sometimes, simplicity evaluating potential resolutions, and selecting the answer
is the key. Overcomplicating the solution to a paradox that offers the most effective explanation.

Resolving Paradox in an Argument 33


Application Drill Passage 2
Read the following question and try to solve it on your own. In In a secluded village, residents consume a local fruit known for its
case of confusion, refer to the explanations. exceptional health benefits. Surprisingly, the village has a higher-
Passage 1 than-average rate of health issues compared to neighbouring
communities. Local health experts claim that the very fruit
Omnifilm, a large film production studio, will release its next
celebrated for its health advantages might be contributing to the
major movie, FastCar, six months from now. Although OmniFilm
generally begins advertising six months before the release of a health problems in this village.
major film, the studio plans to initiate the FastCar ad campaign 3. Which of the following, if true, adds the most complexity
three months from now. to this apparent paradox?
2. Which of the following, if true, best explains OmniFilm’s (a) The village has a unique genetic makeup that reacts
decision to wait three months before beginning the ad differently to the nutrients in the fruit compared to other
campaign? populations.
(a) Many studies show that the benefits of advertising a (b) The village has a thriving fitness culture, with residents
film are maximized when the ad campaign is initiated engaging in regular exercise routines.
no more than six months before a film’s release. (c) Studies show that the health issues in the village are not
(b) Initiating the ad campaign for FastCar in six months directly linked to the consumption of the local fruit.
would unquestionably be less effective than initiating (d) The neighbouring communities have access to
the campaign in three months. advanced medical facilities, providing them with better
(c) Commencing the ad campaign for FastCar would attract healthcare.
public attention away from another OmniFilm movie Sol. Option (a) Unique Genetic Makeup: If the village has a
currently showing in theatres. genetic makeup that responds differently to the fruit, it
(d) Early reviews predict that FastCar will be one of the introduces a genetic factor into the paradox. This could imply
highest-grossing films in OmniFilm’s history that the fruit’s impact varies based on individual genetics,
Sol. Option (a): The issue in the stimulus does not concern adding complexity to the health equation.
advertising initiated more than six months before a film’s Option (b) Thriving Fitness Culture: If the village maintains
release. The paradox is about why FastCar is being a strong fitness culture, it challenges the notion that health
advertised only three months before release, as opposed to issues are solely due to the fruit. This option suggests
the standard six months. that other lifestyle factors, like exercise habits, might
Option (b): This answer choice could be tempting for test be influencing health outcomes, creating a multifaceted
takers who do not read closely. Remember, the stimulus scenario.
states that the ad campaign for FastCar will be released in Option (c) Studies Disassociating Health Issues: This
six months, but advertising will not begin for another three option directly challenges the assumption that the fruit is
months (only three months before the film’s release). This linked to health problems. If studies demonstrate that health
answer choice states that an ad campaign that begins in three issues aren’t connected to fruit consumption, it contradicts
months, as FastCar’s will, will be more effective than one the initial premise and adds a layer of uncertainty to the
that begins in six months (which is when the film opens). paradox.
However, the oddity of this stimulus is not about what would
Option (d) Access to Advanced Medical Facilities: The
happen if the ads started in six months; the paradox concerns
neighbouring communities having better healthcare facilities
why the ad campaign doesn’t begin now, six months before
the film’s release. So this answer choice has nothing to do introduces an external factor. If improved healthcare is
with the facts in the stimulus. influencing health outcomes, it questions whether the fruit
is the primary culprit or if healthcare disparities play a role
Option (c): This is the correct answer. This answer tells us
in the paradox.
that if the ad campaign were to begin now, six months before
the film’s release, it would distract people from another one The most appropriate answer is (c) Studies show that
of the studio’s movies that is currently showing. Since the the health issues in the village are not directly linked to
studio doesn’t want to attract attention away from its current the consumption of the local fruit. This option directly
movie, we now have a reason for the apparent delay in the challenges the assumption that the local fruit is causing
advertising campaign for FastCar. health problems in the village. If studies demonstrate that
Option (d): This answer seems to contribute even further health issues aren’t connected to fruit consumption, it
to the paradox in the stimulus. If FastCar is expected to contradicts the initial premise and adds a layer of uncertainty
be hugely successful, then the studio should be even more to the paradox. This information implies that there might be
willing to begin advertising it as soon as possible. So the other factors contributing to the health problems, making
three-month delay becomes even more counterintuitive. the relationship between the fruit and health issues less
Thus Option (c) is the correct answer. straightforward.

34 Critical Reasoning PW
9 Method and Flaw
in the Reasoning

Method of Reasoning questions prompt you to identify the approach THE FLAW IN THE REASONING
the author takes in presenting an argument. These questions, Flaws in the Reasoning questions closely resemble the Method of
belonging to the First Family of Questions, differ from concrete Reasoning questions, but they come with a key distinction – the
Must Be True questions as they focus on the logical structure rather question stem explicitly states that there is a flaw in the reasoning
than specific details. When tackling Method of Reasoning questions, within the stimulus. This crucial difference simplifies your task
you must rely solely on the information provided in the stimulus because you don’t need to independently assess the validity of
to determine the correct answer. Any answer choice introducing the stimulus; the question stem does this for you. This advantage
elements or situations not found in the stimulus is incorrect. The allows you to pinpoint the flaw in the argument before delving
stimulus in a Method of Reasoning question can showcase either into the answer choices. Even if you initially overlooked the flaw,
sound or flawed reasoning. The questions vary in format, with the the question stem guides you to reassess the argument and spot
stem directing your attention to the method, technique, strategy, the error. When the question stem signals the presence of a flaw, it
or process employed by the author in constructing the argument. might use phrases like “the reasoning is flawed” or “the argument
The aim is to grasp the overarching logical organization of the is vulnerable”, making it clear that a logical misstep exists. This
argument rather than the specific content. explicit identification of a flaw acts as a beacon, directing your
In essence, these questions challenge you to understand how attention to areas of weakness within the argument. It serves as
the author builds and presents their case. This involves recognizing a valuable tool, ensuring that you approach the answer choices
patterns, identifying reasoning structures, and discerning the with a keen awareness of the underlying error in the reasoning
author’s approach to constructing a persuasive argument. Your presented. In essence, Flaws in the Reasoning questions provide
task is to delve into the abstract nature of the argument, using your you with a roadmap to navigate the argument’s pitfalls. You’re
understanding of logic and reasoning to determine the most fitting prompted to recognize and understand the flaw before evaluating
answer choice. To excel in the Method of Reasoning questions, potential answer choices. This strategic guidance enhances your
it’s crucial to navigate the abstract terrain, distinguishing between ability to dissect the reasoning structure and identify common
well-founded and flawed reasoning. Whether the author employs errors, such as logical fallacies or unsupported assumptions.
a deductive approach, draws valid inferences, or commits logical The process involves leveraging the information provided in the
fallacies, your focus remains on the logical architecture of the question stem to your advantage. By acknowledging the existence
argument. Remember, the stimulus is your sole guide, and any of a flaw, you gain insights into the argument’s vulnerabilities.
external knowledge or assumptions should be set aside. This recognition sets the stage for a more targeted and effective
evaluation of the answer choices, allowing you to select the option
THE QUESTION STEMS FROM THE METHOD that most accurately addresses the identified flaw.
OF REASONING
Questions Stem from Flaws in the Reasoning
The following table contains a non-exhaustive list of question
Following are some examples of question stems used for asking
stems that will help you to identify the method of reasoning
questions on the central flaw in the passage.
questions:
“Which of the following most accurately describes a flaw in the
“The method of the argument is to”
argument’s reasoning?”
“The argument proceeds by”
“The reasoning in the argument is most vulnerable to criticism
“The argument derives its conclusion by” because the argument”
“Which of the following describes the technique of reasoning “The reasoning above is flawed because it fails to recognize
used above?” that”
“Which of the following is an argumentative strategy employed “A questionable aspect of the reasoning above is that it”
in the argument?”
Prephrasing in Method and Flaw Questions
“The argument employs which one of the following reasoning
techniques?” The method of Reasoning and Flaw in the Reasoning questions
present a unique challenge as they require abstract thinking,
emphasizing the form of the argument rather than its concrete Caution is warranted for partially true answers—those
details. Navigating these questions becomes intricate because the that capture an aspect of what occurred in the argument but also
answer choices employ abstract terms to describe the argument, introduce additional elements that did not transpire. Consider an
and test makers adeptly manipulate these terms to create answer choice asserting, “The author disagrees with the analogy
unexpected and deceptive options. Students often encounter used by the critic”. To validate this answer, one must locate both
difficulty when none of the answers aligns precisely with their the “disagreement” and the “analogy” in the stimulus. If only
preconceived notions, mainly because the test makers can one or neither can be identified, the answer is incorrect. Even if
encapsulate entire sections of the stimulus in just a word or two. the disagreement is evident, the presence of the analogy in the
Success in these questions hinges on a robust understanding of the stimulus is essential for the answer to be considered correct. This
argument’s mechanics and the ability to discern references within method ensures a rigorous evaluation of each answer choice,
promoting accuracy in identifying the alignment between the
the answer choices.
answer and the actual occurrences in the stimulus.
Prephrasing in Method and Flaw questions involves
grasping the details of the stimulus while also understanding Incorrect Answers in Method and Flaw Questions
its underlying structure. Although you may comprehend the In this section, we’ll delve into specific types of answers related
specifics, the challenge arises when trying to articulate the logical to Method and Flaw questions, expanding our understanding of
organization of the argument. Consequently, each answer may these concepts and introducing an additional wrong answer type:
initially seem implausible, as they primarily relate to the abstract 1. “New” Element Answers: Correct answers for the Method
structure of the argument. To tackle this, it’s crucial to contemplate of Reasoning must align with elements in the stimulus. Any
the argument’s structure before delving into the answer choices. answer introducing something not in the stimulus is incorrect.
While your exact prephrase may not appear among the options Variations of this theme manifest in the wrong answer choices
due to the varied ways an argument can be described, a general, discussed below.
abstract prephrase provides a foundation. A rigorous examination 2. Half Right, Half Wrong Answers: CLAT often crafts
of each answer choice is then necessary to identify if the test answers that start by describing a stimulus occurrence but
makers have paraphrased your prephrase. end by introducing elements not in the stimulus. The rule here
Abstract thinking, inherent in these questions, demands more is straightforward: if an answer is partly wrong, it’s entirely
cognitive effort than concrete thinking, making them challenging wrong.
for most students. The deceptive nature of abstract questions 3. Exaggerated Answers: This type stretches a stimulus
often leads students astray as they grapple with the description situation to an extreme statement unsupported by the
provided by the test makers. Overcoming this challenge requires a stimulus. Be cautious; though extreme language might be
meticulous comparison of the answer choice descriptions with the present, it doesn’t inherently make the answer incorrect.
stimulus. By aligning the abstract prephrase with the nuances of 4. The Opposite Answer: As the name implies, this answer
the answer choices, students can unveil the correct response. This provides the exact opposite of correctness. It’s a clear
approach not only reinforces the importance of understanding departure from what occurred in the stimulus.
the argument’s form but also highlights the necessity of abstract 5. The Reverse Answer: Appealing as it contains familiar
thinking skills in deciphering these intricate Methods and Flaw stimulus elements, the Reverse Answer flips them in the
questions. response. Since the reversed statement doesn’t align with
the stimulus, it’s deemed incorrect.
The Fact Test in Method and Flaw Questions Understanding incorrect answer choices aids future
In Method of Reasoning and Flaw in the Reasoning questions, a preparation. Familiarity with CLAT’s recurrent reasoning methods
useful strategy akin to the Fact Test from Must Be True questions and the language employed to describe them enhances readiness
can be applied to eliminate incorrect answers. The Fact Test in for encountering similar questions. Studying all Methods of
these questions operates on a simple principle: Reasoning and Flaws in the Reasoning answers, both correct
If an answer choice describes an event or element that did and incorrect, proves valuable. Maintaining a list of different
argument types encountered is a practical strategy. Recognizing
not occur in the stimulus, then that answer is incorrect.
that a wrong answer to one question could be correct for another
Test makers often craft tempting but incorrect answer choices builds a robust foundation. During review, contemplating the type
containing elements not present in the stimulus. To navigate this, of argument needed for each incorrect choice sharpens the ability
it’s crucial to select the answer that accurately describes what to discern diverse argument structures.
unfolded in the stimulus. For instance, if an answer states, “The
argument accepts a claim based on public opinion of the claim”, The Value of Knowing Common Errors of Reasoning
every component of the answer must find its counterpart in the In logic, there are more ways for arguments to go wrong than to go
stimulus. Identifying where the author “accepts a claim” and right. The creators of the CLAT, being human, tend to repeat certain
doing so “based on public opinion of the claim” must both be mistakes in the arguments they present. Understanding these common
verifiable in the stimulus. Any part of an answer that cannot be errors gives you a clear advantage. Think of it like examining a tree
identified in the stimulus renders that answer incorrect. in a forest: Must Be True questions focus on one tree, analyzing its

36 Critical Reasoning PW
details, while Method or Flaw questions take a step back, looking at Common Phrases in Answer Choices:
the whole forest, and understanding the general structure. “Bases a conclusion on claims that are inconsistent with
This knowledge serves you in two crucial ways: each other”.
1. Identifying Errors in the Stimulus: Learning the common “Introduces information that contradicts the conclusion”.
mistakes made by authors enables you to quickly spot errors 3. Exceptional Case/Overgeneralization
in the argument. This accelerates your progress through Example: “Two of my friends were shortchanged at that
answer choices, providing confidence and efficiency. Without store. Therefore, everyone gets shortchanged at that store”.
this insight, others might struggle, working more slowly and This error involves drawing a broad conclusion from a small
less confidently. number of instances. Just because two friends had a bad
2. Recognizing Common Error Types in Answer Choices: In experience doesn’t mean everyone will.
Flaws in the Reasoning questions, certain answer types are Common Phrases in Answer Choices:
frequently repeated by the test makers. While these choices “Supports a general claim based on a single example”.
could sometimes be correct based on the reasoning used in “The argument draws a broad conclusion from a small sample
the stimulus, more often, they are standard wrong answers. of instances”.
Being familiar with these choices grants you an advantage, Errors in Assessing the Force of Evidence
helping you swiftly assess their correctness. For instance,
1. Lack of evidence for a position is taken to prove that position
attacking the source of an argument instead of the argument is false
itself is a recurring theme. Knowing this allows you to make
Example: “The White House has failed to offer any evidence
quick determinations during the exam.
that they have reached a trade agreement with China.
Understanding these aspects clarifies why effective test Therefore, no such agreement has been reached”.
preparation is beneficial. The more you comprehend the exam Just because there’s no evidence presented doesn’t mean the
beforehand, the less time you waste pondering these issues during opposite is true. A lack of confirmation doesn’t automatically
the actual test. Knowing the most common flawed reasoning disprove the claim.
patterns on the CLAT is immensely advantageous. Common Phrases in Answer Choices:
Common Errors of Reasoning Explained “Treats failure to prove a claim as constituting a denial of
Let’s break down the errors in reasoning related to evidence in a that claim”.
more straightforward manner. “Taking a lack of evidence for a claim as evidence
Errors in the use of Evidence: undermining the claim”.
2. Lack of evidence against a position is taken to prove that
1. General Lack of Relevant Evidence for the Conclusion
position is true
Example: “Some critics claim that scientific progress has
Example: “There has been no evidence given against the
increased the polarization of society and alienated large existence of God, so God must exist”.
segments of the population. But these critics are wrong
The absence of evidence against a position doesn’t prove that
because even a cursory glance at the past shows that society
the position is true. Lack of disproof doesn’t equal proof.
is always somewhat polarized and some groups are inevitably
Common Phrases in Answer Choices:
alienated”.
“Treating the failure to prove a claim to be false as if it is a
In this case, the author provides irrelevant evidence to counter
demonstration of the truth of that claim”.
the claim about scientific progress. They argue that society
3. Some evidence against a position is taken to prove that the
has always been polarized, which doesn’t directly address
position is false
the impact of scientific progress on polarization.
Example: “Some historians claim that a lengthy drought
Common Phrases in Answer Choices: preceded the fall of the Aztec empire. But we know from
“The author cites irrelevant data”. Aztec writings that in at least one year during the supposed
“It fails to give any reason for the judgment it reaches”. drought, there was minor flooding. Thus, the claim that there
2. Internal Contradiction was a lengthy drought before the fall of the Aztec empire is
false”.
Example: “Everyone should join our country club. After
Introducing evidence against a position weakens it but doesn’t
all, it’s an exclusive group that links many of the influential
necessarily disprove it. A single instance of flooding doesn’t
members of the community”.
negate the possibility of an overall drought.
The contradiction arises when the author urges everyone to
Common Phrases in Answer Choices:
join an exclusive group. “Exclusive” implies some level of
exclusion, contradicting the idea that everyone should be “It confuses weakening an argument in support of a given
part of it. conclusion with proving the conclusion itself to be false”.

Method and Flaw in the Reasoning 37


4. Some evidence for a position is taken to prove that position premise and conclusion essentially say the same thing. In simpler
is true. terms, it’s like saying, “This is the best because it is better than
Example: “We know that the defendant was in the vicinity all the others”. The issue here is that the statement supporting the
of the robbery when the robbery occurred. Therefore, the conclusion is, in essence, the conclusion itself. Example: “I must
defendant is guilty of the robbery”. be telling the truth because I’m not lying”. In this example, the
Having evidence that supports a position doesn’t make it statement “I’m not lying” is used to support the claim of telling
undeniably true. Other factors must be considered. the truth, but it doesn’t provide any independent evidence or
reasoning. The argument goes in a circle, as the claim of not lying
Common Phrases in Answer Choices:
is dependent on the assumption that the person is telling the truth.
“The argument takes facts showing that its conclusion could
be true as proof that the conclusion is indeed true”. Common Phrases in Answer Choices:
In essence, these errors highlight how evidence can be “It assumes what it seeks to establish”.
misused or misunderstood, leading to flawed reasoning in “Presupposes the truth of what it sets out to prove”.
arguments. “The argument assumes what it is attempting to demonstrate”.
Circular reasoning is like a loop where the author uses a
SOURCE ARGUMENT
statement to prove something, and that something is then used to
Let us try to simplify the concept of ad hominem fallacy: Also prove the initial statement. It’s a bit like saying, “I’m right because
known as an ad hominem, this type of flawed argument attacks I’m right”. The argument lacks independent support or evidence,
the person (or source) making a claim rather than addressing relying on a cycle that doesn’t lead to any real conclusion. Think
the actual argument they present. It’s important to note that on of circular reasoning like a person saying, “I’m the best chef
the CLAT, the focus is solely on the structure of arguments, because my cooking is better than anyone else’s”. When asked
and attacking a person’s character or motives is not considered why their cooking is better, they might simply respond, “Because
a valid approach. The correct way to critique an argument is by I’m the best chef”. It’s a loop without any solid basis or external
challenging the reasoning and evidence presented. Example: support.
“The anti-smoking views expressed by Senator Smith should be
ignored. After all, Smith himself is a smoker!” In this example, Errors of Conditional Reasoning
instead of addressing the arguments against smoking, the focus Mistaken Negation and Mistaken Reversal are errors related
is on attacking Senator Smith personally because of his smoking to conditional reasoning. It involves confusing the necessary
habit. This is an ad hominem fallacy. condition with the sufficient condition. Remember, the necessary
condition is something essential for an outcome, while the
Forms of Source Arguments
sufficient condition is enough to guarantee that outcome.
1. Focusing on the Motives of the Source: This occurs when the
1. Mistaken Negation: Mistaking the absence of an occurrence
critique is directed at the intentions or motives of the person
as evidence that a necessary condition for that occurrence
making the argument rather than the argument itself. For
also did not take place. Example: Assuming that if something
example, questioning someone’s motivation for promoting a
necessary for a goal didn’t happen, it’s evidence that the goal
certain idea without addressing the merits of the idea.
itself wasn’t achieved.
2. Focusing on the Actions of the Source: As seen in the
2. Mistaken Reversal: Mistaking something sufficient to
provided example, this form involves attacking the person’s
achieve an outcome for being required to achieve it. Example:
actions or behaviours instead of engaging with the substance
Acting as if something sufficient to create a good leader is
of their argument. It’s a diversion from the core issues being
also necessary for a good leader.
discussed.
Confusing Necessary and Sufficient Conditions:
In Real-World Scenarios: Source arguments, similar
to ad hominem attacks, are commonly used in various Confusing a Necessary Condition for a Sufficient Condition:
settings, especially by children and politicians. In everyday Concluding that something necessary for a goal is sufficient for its
conversations, people might resort to criticizing the person achievement. Example: Treating something necessary for a good
rather than engaging with the actual content of the argument. leader as if it’s sufficient to create a good leader.
Common Phrases in Answer Choices: Confusing a Sufficient Condition for a Necessary Condition:
“It is directed against the proponent of a claim rather than Confusing something sufficient to achieve an outcome with being
against the claim itself”. required for that outcome. Example: Confusing a quality sufficient
for a good leader with something required to create a good leader.
“The attack is directed against the person making the
argument rather than directing it against the argument itself”. Recognizing the Key Terms: When analyzing answer
choices related to conditional reasoning, terms like “sufficient”,
CIRCULAR REASONING “necessary”, “assured” or “required” are crucial. Identifying
Circular reasoning occurs when the author assumes something to these terms helps quickly spot the error in conditional reasoning
be true, which is supposed to be proven, creating a loop where the in CLAT questions.

38 Critical Reasoning PW
Thus, Mistaken Negation and Mistaken Reversal occur when For example:
there’s confusion between what’s necessary and what’s sufficient Politician A: “The platform proposed by my party calls for
for a particular outcome. It’s like mixing up what is crucial and a moderate increase in taxes on those individuals making over
what is enough to make something happen. Recognizing these $20,000 per year, and then taking that money and using it to
errors is essential for understanding and critiquing conditional rebuild the educational system”.
reasoning, and it’s beneficial to look out for specific terms in Politician B: “But what you’re saying is that everyone should pay
answer choices that indicate these mistakes. higher taxes, and so your proposal is unfair”.
Mistaken Cause and Effect Here, Politician B doesn’t engage with the actual proposal
Underneath each item are examples of how the error of reasoning of Politician A. Instead, they distort the argument, claiming that
can be described in answer choices: Politician A suggests everyone should pay higher taxes, which
is not the case. This misrepresentation weakens Politician A’s
1. Assuming a causal relationship based on the sequence of
position, creating a “straw man” that Politician B finds easier to
events: Making the mistake of thinking that just because one
criticize.
thing happens after another, the first thing must be the cause
of the second. Example: Wrongly concluding that because In essence, the “straw man” fallacy is like misquoting
the rooster crows before sunrise, the crowing causes the sun someone and then arguing against the misquote instead of
to rise. addressing their real statements. This misleading tactic can be
spotted by paying attention to how an argument is framed and
2. Assuming a causal relationship when only a correlation
whether it accurately represents the opponent’s position or distorts
exists: Confusing the idea that two things happen together it for convenience.
with the belief that one thing causes the other. Example:
Wrongly thinking that ice cream sales cause an increase in Appeal Fallacies
shark attacks simply because both happen more frequently 1. Appeal to Authorit: This happens when someone tries to
in the summer. convince you of something by using the opinion of an expert,
3. Failure to consider an alternate cause for the effect or but the expert may not know enough about the topic. The
an alternate cause for both the cause and the effect: flaw in this form of reasoning is that the authority may not
Forgetting to think about other possible explanations for have relevant knowledge or all the information regarding
why something happened. Example: Not considering that a situation, or there may be a difference of opinion among
an increase in sunglasses sales and ice cream consumption experts as to what is true in the case. Example: Imagine if
during summer may be because of the hot weather, rather a famous brain doctor says toothpaste is the best. That’s
than assuming one causes the other. nice, but he’s not a tooth expert. Imagine your math teacher
telling you the best way to bake a cake. They might be great
4. Failure to consider that the events may be reversed:
at math, but are they a baking expert? This mistake is like
Mistaking the effect for the cause or vice versa.
trusting an expert in one area to be an expert in another. If
Example: Incorrectly believing that a fever causes an a famous scientist talks about science, cool. But if they talk
increase in body temperature, when, in fact, it’s the other about fashion, maybe not so much. So, if someone says,
way around. “Trust me, I’m an expert”, check if it’s about what they’re
It’s important to note the frequent use of the words “cause” or good at.
“effect” in these descriptions. This repetition occurs because 2. Appeal to Popular Opinion/Appeal to Number: This
these terms are fundamental to expressing issues related to is when people say something is true just because many
causality. So, if you spot a causal reasoning scenario in a people believe it, but that doesn’t always make it right. An
CLAT question, a quick scan for the words “cause” or “effect” appeal to popular opinion does not present a logical reason
in the answer choices might just lead you to the correct one. for accepting a position, just an appeal based on numbers.
Straw Man Example: If most friends say a movie is cool, it doesn’t mean
you’ll like it. Facts matter more than just what many people
This error, known as the “straw man” fallacy, crops up when an think. Just because many people like a song doesn’t mean
author aims to dismantle an opponent’s position not by engaging it’s the best song. Numbers don’t always show what’s true.
with their actual statements but by distorting and weakening This mistake is thinking something is true just because lots
the argument. Imagine building a scarecrow, a “straw man”, of people think so. Imagine if everyone says pizza is gross,
which is then conveniently knocked down. This tactic involves but you love it. What’s true for you is what matters. So, when
misrepresenting the opponent’s stance, making it easier to attack. lots of people agree, it’s nice, but facts are more important.
Often, phrases like “what you’re saying is” or “if I understand 3. Appeal to Emotion: This occurs when someone tries to
you correctly” precede the reconstructed and weakened argument. persuade you by using strong feelings instead of good
Often this error is accompanied by the phrase “what you’re saying reasons. Example: If you tell a police officer not to give you
is” or “if I understand you correctly”, which are used to preface a ticket because you’ve had a tough month, it’s playing on
the refashioned and weakened argument. emotions, not explaining why you shouldn’t get a ticket. If

Method and Flaw in the Reasoning 39


you ask for a toy with puppy eyes, you’re using feelings, not whole life outside parties is also fun and exciting just because of
reasons. That’s like this mistake-using emotions to convince the pen is a mistake.
instead of good explanations. This error is when someone
wants you to agree because they make you feel sad, happy, or Error of Composition
scared. If you want a toy, it’s better to explain why you should It’s like thinking something true for one piece is true for the whole.
have it, not just look sad. So, when someone says, “Feel bad Example: If one slice of pizza is tasty, thinking the whole pizza
for me, so do what I want”, be careful. Good reasons matter is just as good. Imagine you say, “Every homework assignment I
more than just feelings. finish is excellent. Therefore, my school year is excellent”. But,
wait! Just because each assignment is excellent doesn’t mean
Survey Errors your whole school year is the same. It’s like saying, “Every puzzle
Imagine you want to know which ice cream flavor is the most piece is blue. Therefore, the whole puzzle is blue”. Nope, some
popular among your friends. So, you decide to ask only those pieces might be red or green.
friends who attend the same sports club as you. But, whoops!
You forgot that your sports club friends are all big fans of mint Error of Division
chocolate chip ice cream. If you conclude that mint chocolate chip It’s like assuming everyone in a group is the same as the whole
is the most popular flavour based on this group, you’ve just made group. Example: If your friend is great at soccer, thinking
the first mistake in surveys. everyone in their team is also great. Imagine you say, “The entire
1. Biased sample: It’s like asking only a specific group and school is fantastic. So, every student must be fantastic too”. But
thinking they represent everyone. Example: Imagine you hold on! Just because the whole school is fantastic doesn’t mean
want to know if students like a new cafeteria menu. If you every student is. It’s like saying, “This cake is sweet. Therefore,
only ask students who love pizza and ignore others, you every slice of the cake is sweet”. Well, maybe one slice has extra
won’t get a fair idea. Surveys can go wrong if you ask the sugar, and another has less.
wrong people. In 1936, a magazine sent out voting ballots In a nutshell, these errors are like thinking one puzzle piece
mostly to rich folks with phones and cars. Surprise, surprise, represents the whole picture or assuming a tasty slice means the
they predicted the rich Republican would win, but the actual entire pizza is delicious.
winner was the Democrat. Why? Because they asked the
wrong crowd and those who answered were the ones who UNCERTAIN USE OF A TERM OR CONCEPT
felt like it. Imagine if only the kids who love recess get to
Imagine you’re talking about your favourite game, and you use the
vote on the school menu. The result might be all about pizza
word “play” to describe both enjoying the game and manipulating
and cookies, missing what others want.
pieces. Confusing, right? Similarly, in an argument, using a term
2. Improperly Constructed Questions: If questions are in different ways can be like saying “apple” to mean both a fruit
tricky or make you scratch your head, answers might not and a tech company. It’s a mix-up that confuses everyone.
make sense. Example: If a friend asks, “Don’t you think
everyone should agree to clean up after lunch?” You might Error of Ambiguous Term Usage
feel tricked because not everyone might agree. Surveys can It’s like using one word to mean different things, making the
mess up if questions are a puzzle. If someone asks, “Should argument unclear. Example: If you say, “I love playing chess; it’s
the school close early?” but you think it’s already closing a strategy game. Companies should also play fair”. Here, “play”
late, the answer is confusing. Imagine a friend asking, “Do means different things. Imagine saying, “Building a strong team is
you think none of us may want more homework?” That’s like like building a sturdy house; both need a solid foundation”. Now,
saying, “Do you think all of us want more homework?” It’s if you suddenly say, “But in business, building relationships is the
confusing! If questions aren’t clear, answers won’t be either. foundation”, it’s confusing. Here, “building” shifts from a literal
3. Inaccurate Responses: Sometimes, people don’t tell the sense (house) to a metaphorical one (business relationships).
truth in surveys. Example: If a teacher asks, “Did everyone In a nutshell, it’s like talking about a cool movie and
finish their homework?” Some might say yes, even if they saying, “The plot was thrilling, and the land for sale nearby is
didn’t. Surveys rely on people being honest. But, surprise,
also thrilling”. Wait, what? Using “thrilling” for a story and land
they’re not always. Imagine a survey asking, “How many
doesn’t make sense. Similarly, in an argument, using a term in
times do you eat veggies each week?” Some might say every
different ways can leave everyone scratching their heads.
day sounds good, but maybe it’s only on pizza day. It’s like
asking, “How much do you love school?” Some might say
FALSE ANALOGY
a lot because they want to look good, even if they secretly
wish for more playtime. Imagine comparing choosing a movie to choosing a career. It’s a
bit like saying, “Just as picking a comedy makes for a fun movie
ERRORS OF COMPOSITION AND DIVISION night, the best way to find a job is to spin a wheel and see where it
Imagine you have a magical pen, and every time you use it, your lands”. Well, that comparison is a bit off, right? That’s what we call
writing becomes fun and exciting. Now, let’s say you use that pen a False Analogy. It’s like saying two things are similar when, in
only at parties. So, logically, every party you attend becomes fun reality, they’re quite different. Example: Saying, “Just like adding
and exciting because of the pen. But, oops! Thinking that your sugar to tea sweetens it, adding salt to your coffee will make it

40 Critical Reasoning PW
tasty”. Um, no, that’s not how flavours work. Picture this: “Just as (past financial performance) have a certain characteristic
a chef needs various ingredients to cook a gourmet dish, a writer (profitability), it doesn’t mean the entire company will
needs diverse words to craft a masterpiece”. Now, if you suddenly exhibit the same characteristic in the future.
say, “So, just mix random words for a great novel”, that’s a bit of 2. Argument: Over the past decade, the crime rate in City X
a stretch. Cooking and writing are different arts; the analogy fails. has steadily decreased. Therefore, implementing a new set
It’s like comparing swimming in the ocean to navigating a board of strict regulations on businesses will further contribute to
game. Saying, “Just as a swimmer battles waves, strategizing in the reduction in criminal activities in the city. Identify the
Monopoly is intense”. Hold on, those situations are worlds apart! flaws in the argument presented above.
Similarly, in an argument, comparing unrelated things is like (a) Ad Hominem
trying to fit a square peg into a round hole. It just doesn’t work!
(b) Hasty Generalization
TIME SHIFT ERRORS (c) Correlation/Causation Fallacy
Imagine if someone said, “I’ve always had chocolate ice cream (d) Straw Man Fallacy
on my birthday, so I’ll have it this year too”. That’s a bit like Sol. Option (a) Ad Hominem (Incorrect): Ad Hominem attacks
assuming the future based on the past, and it’s called the “Fallacy the person making the argument rather than addressing the
of False Analogy”. Assuming things will stay the same over time. argument itself. The argument here doesn’t involve personal
Example: Believing, “The weather has always been sunny on attacks but rather draws a conclusion based on a specific
picnics, so our picnic next year will be sunny too”. Let’s dive claim.
into this. Imagine if your friend said, “I’ve never missed the bus Option (b) Hasty Generalization (Correct): Hasty
before, so I won’t miss it tomorrow”. Well, that’s not a guarantee. generalization occurs when a conclusion is drawn from
It’s like saying, “I’ve always aced math quizzes, so I’ll ace this insufficient evidence. In this argument, the reduction in
one too”. Past success doesn’t guarantee future victories. crime rate is attributed to a specific cause (strict regulations
Illustration on businesses) without considering other factors that may
have contributed to the decline.
1. Argument: The company has been profitable for the past
five years, and its stock value has consistently increased. Option (c) Correlation/Causation Fallacy (Incorrect):
Therefore, it is evident that the company will continue to While the argument does suggest a correlation between
experience financial success in the coming years. Identify implementing strict regulations and reduced crime, it doesn’t
the flaw in the argument presented above. explicitly claim that one causes the other. This flaw is not
(a) Circular Reasoning as evident in the given argument.
(b) False Analogy Option (d) Straw Man Fallacy (Incorrect): A straw man
fallacy involves misrepresenting an opponent’s argument to
(c) Error of Composition
make it easier to attack. In this case, the argument doesn’t
(d) Fallacy of False Cause create a distorted version of an opposing view but rather
Sol. Option (a) Circular Reasoning (Incorrect): Circular relies on an unsupported assumption.
reasoning involves restating the argument differently The primary flaw in the argument is a hasty generalization.
without providing additional support. In this argument, It assumes that implementing strict regulations on businesses
the conclusion is essentially repeated, but the flaw lies in is the sole cause of the decrease in the crime rate without
assuming past profitability guarantees future success. considering other potential factors. Additionally, the
Option (b) False Analogy (Incorrect): A false analogy argument fails to provide sufficient evidence and overlooks
involves comparing two unrelated things. This argument the complexity of crime rate reduction, making its conclusion
doesn’t compare different situations; instead, it assumes a premature and unsupported.
consistent trend without considering external factors. It’s
not an issue of comparing unrelated items. Application Drill
Option (c) Error of Composition (Correct): This error occurs Read the following argument and solve the question on the central
when a characteristic of a part is incorrectly attributed to flaw. To verify the answer, refer to the explanation provided.
the whole. In this case, just because the company has been 3. Argument: Proponents of the theory of social utilitarianism
profitable doesn’t mean every aspect (or future performance) hold that the value of human capital should bear an inherent
of the company will be equally successful. relation to its social utility. Although maximizing the value of
Option (d) Fallacy of False Cause (Incorrect): The fallacy human capital is both morally defensible and economically
of false cause involves assuming that because two events praiseworthy, the theory of social utilitarianism has severe
are correlated, one causes the other. While the argument practical limitations. If the price of labour were to become
suggests a correlation between past profitability and future a measure of social utility and not of scarcity, the labour
success, it doesn’t explicitly state that one causes the other. market would suffer significant distortions that may well
Thus, The flaw in the given argument is an error of reduce, and not increase, the current level of human capital.
composition. Just because the parts of the company The argument proceeds by

Method and Flaw in the Reasoning 41


(a) Questioning a proposed strategy by showing that, if than those who lead sedentary lifestyles. The study suggests
implemented, such a strategy could compromise the a strong correlation between physical activity and job
very objectives it is trying to achieve. satisfaction, asserting that employers should encourage and
(b) Criticizing a course of action by showing that, even if facilitate exercise programs for their employees. However,
morally defensible, the result does not always justify the critics argue that the study fails to establish a clear cause-
means necessary to achieve it. and-effect relationship and overlooks other factors that
(c) Criticizing a strategy by suggesting that there is an contribute to job satisfaction, such as work environment and
alternative way of achieving its proposed advantages job responsibilities. Identify the central flaw in the argument
without risking several serious disadvantages. presented.
(d) Conceding that a social policy may have certain ethical (a) The argument relies on circular reasoning by assuming
advantages that are ultimately outweighed by the that individuals who engage in regular physical exercise
impossibility of putting such a policy into effect. are inherently more satisfied with their jobs, without
providing evidence for the causal relationship between
Sol. Option (a) This option accurately reflects the structure of the exercise and job satisfaction.
argument. The passage questions the strategy of tying the (b) The argument makes a false analogy by comparing job
value of human capital to its social utility by demonstrating satisfaction to physical exercise without considering
that if this strategy is implemented, it could lead to fundamental differences between the two, leading to an
significant distortions in the labour market, compromising invalid conclusion.
the objectives of maximizing the value of human capital.
(c) T h e a rg u m e n t c o m m i t s t h e f l a w o f c a u s a l
Option (b) While the argument mentions that maximizing
oversimplification by attributing job satisfaction solely
the value of human capital is morally defensible, the primary
to physical exercise and neglecting other significant
criticism is about the potential negative consequences
factors that contribute to overall job satisfaction.
and distortions in the labour market, not about the means
necessary to achieve the result. (d) The argument improperly relies on the authority of the
Option (c) The argument does not propose an alternative study’s researchers, assuming their conclusions about
strategy. Instead, it focuses on the drawbacks and practical the correlation between exercise and job satisfaction
limitations of the theory of social utilitarianism, without are unquestionably valid.
suggesting an alternative approach. Sol. Option (a) This option is incorrect as the argument does not
Option (d) The argument does not concede ethical exhibit circular reasoning. The flaw lies in the assumption
advantages; rather, it emphasizes the severe practical of a causal link without sufficient evidence.
limitations of the theory of social utilitarianism. The focus Option (b) This option is incorrect as the argument does not
is on the potential negative consequences rather than the make a false analogy between job satisfaction and physical
ethical advantages being outweighed by implementation exercise. The flaw is related to causal reasoning.
difficulties. Option (c) This is the correct answer. The argument
In summary, Option (a) accurately captures the essence of oversimplifies the relationship between physical exercise
the argument, making it the correct answer. The argument and job satisfaction, neglecting other influential factors.
questions the proposed strategy and demonstrates that its Option (d) This option is incorrect as the flaw is not related
implementation could compromise its objectives. to an inappropriate appeal to authority. The issue lies in the
4. Argument: In a recent study, researchers concluded that reasoning behind the causal relationship presented in the
individuals who engage in regular physical exercise are study.
more likely to experience higher levels of job satisfaction Thus Option (c) is the correct answer.

42 Critical Reasoning PW
10 Parallel Reasoning

Parallel Reasoning questions require you to find an answer In the same way, on tests like the CLAT, they give you a little
choice that follows a similar method of argumentation as the one story or argument, and they ask you to pick out the story that’s
presented in the stimulus. This task involves first understanding built the same way among a bunch of options. It’s not about the
how the author in the stimulus constructs their argument and then specific things they’re talking about in the stories but about how
identifying a match among the answer choices. Imagine you’re the stories are structured. For example, if your friend’s story
looking for a friend in a crowd, and each person represents an was about how eating healthy food makes people happy, and the
answer choice. Your task is to find the person (answer choice) reason was that it makes them feel good, a similar story might
who walks, talks, and behaves most similarly to your friend (the be about how exercising makes people happy, with the reason
stimulus). Now, let’s break it down. In the stimulus, the author uses being that it also makes them feel good. Even though the topics
(healthy food and exercising) are different, the way the stories
a specific way to build their argument, like pointing out causes
are built is the same—one thing leads to another. So, in parallel
and effects or making analogies. Your job is to recognize this
reasoning, you’re like a detective looking for stories that have the
approach. It’s like understanding your friend’s distinct features same logical structure. It’s not about the details or the characters;
in the crowd—their hairstyle, clothing, or unique gestures. Once it’s about how the ideas are connected. It’s a bit tricky because
you’ve grasped the method of argumentation in the stimulus, you the words might be different, but the way everything fits together
scan the answer choices like searching the crowd. Look for the is what you’re matching. It’s like finding a puzzle piece that fits
one that echoes the same reasoning structure. It’s akin to finding just right—not the same picture, but the same shape. That’s what
the person who mirrors your friend’s characteristics—the same parallel reasoning is all about!
hairstyle, clothing style, or distinctive gestures. Let us try to get more clarity with the help of a basic example
PARALLEL REASONING QUESTION STEMS Original Story: Sarah studies every day, and she gets good
grades. So, studying helps her do well in school.
“Which of the following is most closely parallel in its reasoning Parallel Story: Tom practices basketball every day, and he
to the reasoning in the argument above?” performs well in games. So, practising helps him do well in sports.
“Which of the following exhibits a pattern of reasoning most Even though the topics are different (Sarah studies for
similar to that exhibited by the argument above?” school, Tom practices basketball), the structure is the same. In
both stories, doing a certain thing (studying or practising) leads to
“Which of the following arguments is most similar in its logical success (good grades or good performance). That’s what parallel
features to the argument above?” reasoning is like—finding stories or arguments that have the same
“Which of the following arguments is most similar in its pattern logical shape, even if the details are different. It’s a bit like saying,
of reasoning to the argument above?” “If this story makes sense, then this other one, even if it’s about
something else, should make sense in the same way”.
“The structure of the reasoning in the argument above is most
parallel to that in which of the following?” HOW TO SOLVE PARALLEL REASONING
“Choose the answer that exhibits reasoning most analogous to QUESTIONS?
the pattern seen in the passage”. Let us try to come up with a step-by-step process to easily tackle
questions on parallel reasoning:
“Identify the response that follows a line of reasoning most
closely resembling that of the given argument”.  Step 1: Uncover the Original Argument: Imagine you’re a
detective entering a crime scene. Similarly, read the argument
“Whose approach to reasoning is most akin to that employed in with a keen eye. Identify the main point—the heart of the
the argument above?” case—and the reasons supporting it. In our case, it’s like
discovering the motive behind a crime.
Understanding the Meaning of Parallel Reasoning  Step 2: Break Down the Argument (Identify Key Clues):
It is like finding a twin for an argument. Imagine you have a Picture the argument as a puzzle. Identify the crucial pieces—
friend, and you’re both telling stories. Now, let’s say your friend characters (John), actions (exercising), outcomes (staying
tells a story, and you want to find another story that’s similar in healthy) and the connections. Just like a detective analyzes
how it’s put together, even though the details might be different. evidence, focus on the critical clues that construct the logic.
 Step 3: Grasp the Logic (Ignore the Details): Think of the Sol. Option (a): This argument commits a correlation-causation
argument as a musical composition. Forget the specific notes fallacy. It assumes that because two things happened
(John, exercising) and concentrate on the rhythm—the logical together, one must have caused the other. This is not a
structure. Understand the cause-and-effect flow without parallel flaw to the original argument, which attacks the oil
getting lost in the details. companies’ motives, not a correlation.
 Step 4: Approach Answer Choices (Explore New Option (b): This argument attacks the credibility of the gas
Arguments): Now, shift your mindset to that of an explorer. station owner based on the overall profits of the oil industry.
Move on to the answer choices, treating them as uncharted It’s parallel to the original as it questions the integrity of a
territories. Look for arguments with a similar logical smaller entity based on the success of a larger related entity.
structure. It’s about discovering new lands that follow the This is a strong contender.
same map. Option (c): This argument introduces a different flaw. It
 Step 5: Match Logical Structures (Pairing Twins): Become suggests that because a plan benefits some more than others,
a logical matchmaker. Read each answer choice and try to it should not be implemented. It doesn’t attack the mayor’s
pair its logical structure with the original argument. It’s like motives or imply any contradiction in the argument.
finding a pair of shoes that not only look good but also have Option (d): This argument attacks the board’s decision
a matching structure—cause leading to effect. by pointing out their self-interest in the merger. It’s very
 Step 6: Eliminate Wrong Choices: Channel your inner similar to the original as it questions the motives behind a
detective. Eliminate choices that deviate from the original decision, undermining the claim of acting in the best interest
logical structure. It’s like ruling out suspects based on of all shareholders. This is a strong parallel to the original
inconsistencies or alibis that don’t align with the crime scene. argument.
 Step 7: Choose the Best Match (Make the Final In summary, options (a) and (c) can be eliminated as they
Discovery): Imagine you’re the jury in a courtroom. Select exhibit different flaws compared to the original argument.
the answer that stands trial and closely mirrors the logic of Options (b) and (d) are closely related, but (d) is a better
the original argument. It’s like choosing the most convincing match as it directly attacks the self-interest of the decision-
case based on the evidence presented. makers, aligning with the original argument’s pattern.
Therefore, the correct answer is (d).
Illustration
Passage 2
Passage 1
The government claims that the construction of a new highway
Oil companies argue strenuously that no further restrictions will significantly reduce traffic congestion and boost economic
should be placed on offshore drilling due to our country’s need for development in the region. However, considering the government
energy resources, and the possible serious consequences if new officials’ close ties to the construction companies awarded the
energy reserves are not located and explored now. Of course, the project, it’s clear that their primary motivation is personal gain
vast sums of money the oil companies stand to reap as a result of rather than the public interest.
these drilling efforts completely invalidates any such arguments.
2. Which of the following arguments exhibits a pattern of
1. The questionable pattern of reasoning above is most similar reasoning most similar to that in the passage above?
to which of the following? (a) Local businesses argue that the installation of new
(a) Everyone in town was wearing boots yesterday, and parking facilities will lead to increased customer
everyone in town was also carrying an umbrella. Based traffic. Yet, since many of these businesses have
on this evidence it is obvious that wearing boots causes financial connections with the construction companies
one to carry an umbrella. undertaking the parking projects, it’s evident that their
(b) The owner of a local gas station claims to be losing main goal is personal profit.
money, but everyone knows that the oil industry is (b) Environmentalists advocate for the expansion of public
earning record profits, which undoubtedly proves that parks, asserting it will enhance the community’s well-
the gas station’s owner must be lying. being. However, some of these environmentalists have
(c) The mayor has argued strenuously in favor of a financial ties to landscaping companies involved in the
government housing subsidy. Since such a move would park projects, revealing their real intention of personal
benefit some town residents more than others, this plan enrichment.
should not be implemented. (c) School administrators claim that implementing a
(d) Although the board claims that their vote in favour of new curriculum will significantly improve student
the merger is in the best interest of all shareholders, the performance. Yet, given the administrators’ financial
fact that the merger will be most beneficial to the board’s relationships with textbook publishers promoting
members themselves shows that other shareholders the new curriculum, it’s apparent that their primary
should object to the board’s decision. motivation is personal gain.

44 Critical Reasoning PW
(d) Health officials propose the construction of a new security technology company poised to profit from the
hospital to address the growing healthcare needs of the proposed security upgrades.
community. Examining the officials’ close associations (b) Environmentalists are calling for enhanced protection
with the construction firms set to build the hospital, it’s of local wildlife habitats to preserve biodiversity.
clear that their main objective is personal profit rather However, a closer look reveals that several of these
than the public’s well-being. environmentalists have ties to a consulting firm that
Sol. Option (a): The argument commits a similar flaw by attacking specializes in habitat restoration, indicating potential
the motives of those supporting a project, suggesting financial motivations.
personal profit motives due to financial connections. This (c) Employees are advocating for the introduction of flexible
aligns with the original argument’s pattern. working hours, arguing that it would improve work-life
Option (b): The argument parallels the original as it questions balance. Investigation reveals that a significant number
the motives behind supporting a project, implying personal of these employees have connections to companies
enrichment due to financial ties with companies involved. selling productivity software designed for remote work.
Option (c): This argument introduces a different flaw, (d) Citizens are pushing for stricter regulations on air
focusing on a new curriculum and financial ties. It pollution, citing health concerns. Further scrutiny
doesn’t align with the original argument, which centres on exposes that many of these citizens have investments
construction projects. in companies developing eco-friendly technologies,
Option (d): This argument closely mirrors the original as suggesting a financial interest in supporting stringent
it questions the motives behind supporting a construction environmental regulations.
project, suggesting personal profit interests due to close Sol. Option (a): The argument shares a parallel pattern as it
associations with construction firms. This is the correct questions the motives of those supporting a policy, implying
parallel reasoning. financial interests due to connections with companies that
In conclusion, Option (d) is the correct answer as it exhibits stand to benefit.
a pattern of reasoning most similar to the original argument,
Option (b): This argument introduces a different context,
both involving construction projects and questioning the
focusing on wildlife habitat protection and financial
motives based on financial associations.
connections. It does not align with the original argument’s
Passage 3 emphasis on police presence.
Many citizens argue that increasing the number of police officers Option (c): The argument shifts the focus to flexible working
on the streets is crucial for reducing crime rates. However, when hours and financial ties to software companies. It does not
examining the backgrounds of those advocating for this policy, parallel the original argument regarding increased police
it becomes apparent that most have financial interests in security presence.
companies that stand to benefit from increased police presence. Option (d): This argument closely mirrors the original as it
3. Which of the following arguments exhibits a pattern of questions the motives behind supporting a policy, suggesting
reasoning most similar to that in the passage above? financial interests in companies related to the proposed
regulations. This is the correct parallel reasoning.
(a) Parents are urging the school board to implement
stricter security measures, emphasizing the importance In conclusion, Option (d) is the correct answer as it exhibits
of safeguarding students. Yet, upon investigation, it is a pattern of reasoning most similar to the original argument,
revealed that many of these parents own shares in a both involving policy support and questioning the motives
based on financial interests.

Parallel Reasoning 45
Practice Questions on CLAT Pattern

PASSAGE 1 the challenges and concerns associated with its


A healthy loss and damage (L&D) fund, a three-decade-old implementation.
demand, is a fundamental expression of climate justice. The (b) The passage mainly emphasizes the commitment of
L&D fund is a corpus of money and technologies that will be developed countries to the L&D fund and the significant
replenished by developed countries and used by the rest to contributions they have made to address climate change
respond to the more unavoidable effects of climate change. On the effects.
first day of the COP28 climate talks underway in the United Arab (c) The passage primarily focuses on the success of COP28
Emirates (UAE), representatives of the member-states agreed to climate talks and the diplomatic achievements of its
operationalise the L&D fund. The announcement was dearly won: Emirati president.
at the end of the COP27 talks in Egypt last year, member-states (d) The passage mainly discusses the role of the World Bank
agreed to launch such a fund, thanks largely to the steadfast efforts in hosting the L&D fund and the conditions it needs to
of the G-77 bloc of countries plus China, led by Pakistan. Four fulfil for effective fund management.
meetings of the Transitional Committee (TC) were to follow to 2. What assumption underlies the concerns raised in the passage
determine how its money would be disbursed. But the issues in about the operationalization of the Loss and Damage (L&D)
the TC-4 meeting, which spilt over into an ad hoc TC-5 meeting fund?
as well, highlight how the newly operationalised fund while
(a) The assumption that the commitments made by
signalling optimism at COP28 and a diplomatic victory for its
developed countries to the L&D fund are insufficient
Emirati president, has crucial issues.
and may not be periodically replenished.
First, it will be hosted by the World Bank for an interim
(b) The assumption that the World Bank’s stewardship of the
period of four years and will be overseen by an independent
L&D fund may face challenges and conditions related
secretariat. The Bank is expected to charge a significant overhead
to transparency may not be adequately met.
fee. Developing countries resisted this proposition at first before
yielding at the TC-5 meeting, in exchange for some concessions. (c) The assumption that COP28 climate talks were
Second, while some countries have committed amounts to the unsuccessful in addressing the fundamental issues
fund—from $10 million by Japan to $100 million each by Germany related to climate justice.
and the UAE—whether they will be periodically replenished is (d) The assumption that the G-77 bloc of countries plus
not clear. The committed amounts are also insufficient, totalling China played a negligible role in the launch of the L&D
$450 million (for now) against an actual demand of several fund during the COP27 talks.
billion dollars. This shortfall, though it is premature to deem it 3. Based on the information provided in the passage, what
so, comes against the backdrop of developed countries missing can be reasonably inferred about the concerns raised by
their 2020 deadline to mobilise a promised $100 billion in climate developing countries during the COP28 climate talks?
finance and managing to deliver only $89.6 billion in 2021. Next, (a) Developing countries were primarily concerned about
the contributions are voluntary even as every country has been the financial commitments made by developed countries
invited to contribute. Finally, the World Bank will have to meet to the Loss and Damage L&D fund.
some conditions on managing the fund, including a degree of (b) Developing countries successfully resisted the idea of
transparency it has not brooked so far, and submit a report to the hosting the L&D fund at the World Bank during the
Parties to the Paris Agreement. If its stewardship is determined TC-5 meeting.
to be unsuitable, the fund can ‘exit’ the World Bank. The L&D
(c) The commitments made by Japan, Germany, and the
fund’s contents need to be easily accessible to those who need it
UAE to the L&D fund were deemed sufficient by the
most, in a timely fashion, sans pedantic bureaucratic hurdles, and
developing countries.
in sufficient quantities. As things stand, there is little guarantee
that any of these requirements will be met. While the L&D fund (d) The TC-4 meeting during the COP28 talks did not play
is finally online, a lot more needs to be done. a crucial role in determining the operationalization of
the L&D fund.
Source: Finding funds: The Hindu Editorial on COP28 and
the ‘loss and damage’ fund—The Hindu 4. Which of the following represents a paradoxical situation
as discussed in the passage?
1. Which of the following most appropriately describes the
(a) The commitment amounts to the Loss and Damage
central theme of the passage?
(L&D) fund are insufficient, yet some developing
(a) The passage primarily discusses the operationalization countries initially resisted financial contributions during
of the Loss and Damage (L&D) fund and highlights the COP28 talks.

46 Critical Reasoning PW
(b) The World Bank will oversee the L&D fund, but it is to view the headline growth number with circumspection is the
expected to charge a significant overhead fee, creating fact that the lynchpin private final consumption expenditure, the
a potential financial burden on the developing countries. single-largest component of demand in the economy, is struggling
(c) The COP27 talks agreed to launch the L&D fund, but the for traction. Growth in private consumption spending slowed
TC-4 meeting highlighted issues that spilt over into an appreciably to 3.1% in the September quarter, from 6% in the
ad hoc TC-5 meeting, creating delays and uncertainties. preceding three-month period, as rural demand remained affected
by the vagaries of a below-average monsoon. The NSO data also
(d) Developed countries missed their 2020 deadline to reveal that the recent economic momentum owes its fillip in large
mobilize $100 billion in climate finance, yet some measure to front-loaded government spending, both in terms of
countries, including Japan, committed amounts to the consumption demand and asset-creating capital investments. The
L&D fund. challenge for policymakers will be to help broaden the growth
5. What is the central flaw in the operationalization of the Loss base to ensure that all boats are lifted equally, both to sustain the
and Damage (L&D) fund, as discussed in the passage? momentum and reduce inequality.
(a) The commitment amounts from developed countries Source: Patchy expansion: The Hindu Editorial on
are insufficient to meet the actual demand for the L&D provisional estimates of GDP for the quarter ending September
fund. 30, 2023—The Hindu
(b) Developing countries initially resisted financial 6. Which of the following accurately describes the structure
contributions during the COP28 talks. of the passage?
(c) The World Bank’s oversight with a significant overhead (a) The passage introduces the latest GDP estimates,
fee creates potential financial burdens on developing discusses the performance of key sectors, highlights
countries. the deceleration in growth, and concludes with a call
(d) The commitment amounts from countries like Japan, for policymakers to address challenges.
Germany, and the UAE are voluntary. (b) The passage begins by analyzing the services sector,
moves on to the performance of manufacturing and
PASSAGE 2 construction, discusses the agriculture sector, and
concludes with a call for broader economic growth.
The latest provisional estimates of GDP, released by the National
Statistical Office, project real economic growth at 7.6%, a slight (c) The passage starts by presenting GDP growth estimates,
deceleration from the 7.8% logged in the preceding three months. delves into the performance of key sectors, analyzes
the slowdown in private consumption, and ends by
Gross Value Added (GVA) across the eight broad sectors of the
emphasizing the need for inclusive growth.
economy also reflected a marginal slowing, with second-quarter
GVA registering a 7.4% expansion, 40 basis points slower than (d) The passage opens with the latest GDP estimates,
the April-June period’s 7.8%. Robust double-digit expansions in evaluates the robust performance of manufacturing and
construction, explores the slowdown in key sectors, and
manufacturing, mining, utilities and construction offset the loss of
concludes with a call for policy intervention.
momentum across the other four sectors and helped ensure that
the year-on-year growth in GVA comfortably exceeded the 7% 7. What can be inferred from the passage regarding the factors
pace for a second straight quarter. Manufacturing, buoyed by a contributing to the recent economic momentum?
favourable base effect due to the contraction in the year-earlier (a) The recent economic momentum is primarily driven by
period, was the strongest performer by registering growth of robust private consumption spending.
13.9%, a nine-quarter high. And construction witnessed its best (b) The growth in the services sector, particularly trade,
showing in five quarters, expanding 13.3%. Of the four other hotels, transport, and communication, is the primary
sectors, the crucial ones of agriculture and the two services sectors factor behind the recent economic momentum.
of trade, hotels, transport and communication, and financial, real (c) Front-loaded government spending, both in terms
estate and professional services saw the pace of growth almost of consumption demand and asset-creating capital
halving from the fiscal first quarter. While year-on-year growth in investments, has significantly contributed to the recent
the agriculture, livestock and fishing sector slowed sharply to an economic momentum.
18-quarter low of 1.2%, the sector also experienced a sequential (d) The expansion in the financial and realty services
contraction for the third straight quarter, underscoring the sector has been the key driver of the recent economic
precarity plaguing large portions of those earning from farming momentum.
and allied activities in the rural hinterland. 8. Which of the following, if true, most strongly supports the
The slowdown in two key components of the services argument that the recent economic momentum might face
economy—trade, hotels, transport and communication saw growth challenges?
slide to 4.3%, from 9.2% in the June quarter, and the expansion in (a) The government plans to further increase spending in
financial and realty services more than halved from the preceding the upcoming quarters.
period to 6%—also merits a closer watch as the post-pandemic (b) Private final consumption expenditure is expected to
rebound in services appears to have lost steam. Adding to the need rebound significantly in the next fiscal year.

Practice Questions 47
(c) The growth in the manufacturing sector is anticipated places, it can be hoped that what has happened in Nepal won’t
to continue at a robust pace. just stay in Nepal. In the aftermath of a verdict that did not go far
(d) Despite the overall economic growth, rural demand enough in India, the news of a queer couple’s marriage in Nepal
remains affected by below-average monsoon conditions. raises hopes of the next step being taken here in the not-too-distant
9. Which of the following statements is supported by the future, now that a model is available in the neighbourhood.
information provided in the passage? After all, the power of a good idea to set an example and
(a) T h e m a n u f a c t u r i n g s e c t o r h a s c o n s i s t e n t l y cross a border has been seen before—for example, with India’s
underperformed in the past nine quarters. Right to Information (RTI) law, which grew from a grassroots
(b) The decline in private final consumption expenditure is movement for greater accountability and transparency in
primarily attributed to government policies. governance to a 2005 Act that subsequently became a model for
other nations in the region. After India, Bangladesh passed its
(c) Construction witnessed its best showing in five quarters,
driven by a favourable base effect. own RTI law in 2008, followed by Sri Lanka in 2016. Ideas that
drive change in the direction of greater equality and freedom can
(d) The services sector experienced a significant rebound
make a larger difference and set off wider ripples and echoes. A
in the September quarter after a prolonged slump.
wedding in Nepal today can become a signpost in South Asia for
10. Which of the following situations is most analogous to the a more equal tomorrow.
growth pattern described for the manufacturing sector in the
Source: Nepal’s first official same-sex marriage: A beacon
passage?
of change for South Asia | The Indian Express
(a) A technology company experiences a surge in profits
due to the release of a highly anticipated product, 11. Which of the following best captures the main idea conveyed
contributing to the overall growth in the technology by the passage?
sector. (a) The Supreme Court’s decision in India to recognize
(b) A retail chain witnesses a decline in sales during a fundamental rights for the LGBTQI+ community,
particular quarter, but the overall retail industry shows although falling short of legalizing marriage equality,
robust growth driven by other major players. has raised hopes for future progress.
(c) A pharmaceutical company achieves record-breaking (b) Nepal’s Supreme Court’s landmark decision to legalize
sales in a quarter, outperforming its competitors in the same-sex marriage has the potential to inspire positive
healthcare sector. change in the region, including India, where recent legal
(d) An automobile manufacturer faces a decline in developments left many disheartened.
production, but the transportation industry, as a whole, (c) Maya Gurung and Surendra Pandey’s official registration
sees increased demand for electric vehicles. of their wedding in Nepal symbolizes a significant step
towards marriage equality and LGBTQI+ rights across
PASSAGE 3 South Asia.
Four months before India’s Supreme Court nudged open the (d) The interconnected world and the power of influential
door, for making available fundamental rights and freedoms to ideas, illustrated by Nepal’s legalization of same-sex
the embattled LGBTQI+ minority but stopped short of legalising marriage, suggest a hopeful trajectory for achieving
marriage equality, a neighbouring country had taken momentous greater equality and freedom in the region.
strides in that direction. In its landmark June 27 verdict, Nepal’s
12. Which of the following, if true, would most weaken the
Supreme Court, responding to a writ petition, legalised same-sex
argument that Nepal’s legalization of same-sex marriage
marriage. The court’s interim order directed the government to
make arrangements to “temporarily register” the marriages of could inspire positive change in the region, including India?
“sexual minorities and non-traditional couples”. The order made (a) Nepal’s decision to legalize same-sex marriage was
Nepal the first South Asian nation—and only the second in Asia met with strong opposition from conservative religious
after Taiwan—to institutionalise marriage equality. groups within the country.
This week, Maya Gurung and Surendra Pandey, who have (b) India’s LGBTQI+ activists have expressed scepticism
been together for 10 years and had a temple ceremony in 2017 about drawing inspiration from legal developments in
with the blessings of their families, became the first queer couple neighbouring countries, emphasizing the unique socio-
in the country to officially register their wedding. Gurung and cultural contexts.
Pandey’s dream of a life together, finally recognised and enabled (c) Despite legal recognition, same-sex couples in Nepal
by the law, is one that will resonate within their country, and also continue to face social stigma and discrimination,
outside it, including in its neighbourhood—in south Asia and in limiting the practical impact of the legalization.
India where the October 17 verdict had left many disheartened. In (d) The Supreme Court’s decision in India, although not
a world that is more interconnected than ever before, and where legalizing marriage equality, has prompted increased
ideas travel much more quickly and strike sparks even in distant public discourse and awareness about LGBTQI+ rights.

48 Critical Reasoning PW
13. In the passage, what role does the statement “This week, in India also view Kissinger as the one man responsible for the
Maya Gurung and Surendra Pandey, who have been together rise of China by drawing it into a long-term partnership with the
for 10 years and had a temple ceremony in 2017 with the United States against the Soviet Union. But Kissinger is more
blessings of their families, became the first queer couple in than the “villain” of the 1971 war and a lifelong “friend of China”.
the country to officially register their wedding” play in the His contributions towards shaping American foreign policy and
overall argument? the evolution of the post-war world in the second half of the
(a) It serves as a counterexample, illustrating that despite 20th century are immense and consequential. As Delhi reflects
legal recognition, same-sex couples in Nepal still face on Kissinger’s celebrated but controversial legacy, his statecraft
challenges. holds lessons for a rising India seeking a larger role in the world.
(b) It functions as supporting evidence, showcasing a The US tilt to Pakistan in 1971 did not involve firing an
tangible and positive outcome of Nepal’s legalization American shot against India, nor did it affect the outcome of the
of same-sex marriage. Bangladesh war. In criticising Kissinger’s China policy, it is easy
(c) It acts as a contradiction, highlighting the disparity to forget that New Delhi was a much bigger political champion of
between legal recognition and the actual experiences China in the 1950s when America was trying to isolate it. Large
of LGBTQI+ individuals. countries like India can’t be obsessed with specific moments
(d) It plays a neutral role, providing factual detail without in history and lose sight of the larger geopolitical dynamic. In
contributing to the argument’s overall impact. Kissinger’s realpolitik, protecting an important Cold War ally—
Pakistan—and turning an adversary like Communist China into
14. What paradoxical situation is presented in the passage?
a strategic partner was driven by a deep consideration for US
(a) The Supreme Court’s decision in Nepal, legalizing interests in the 1970s. That world has now gone. There is now a
same-sex marriage, creates a paradox as it contradicts
robust India-US partnership no longer tied to Pakistan and aimed
the conservative values prevalent in the country.
at securing an Asia that an assertive China has destabilised. As
(b) The passage describes a paradox where the recognition an exponent of power politics, Kissinger had no hesitation in
of same-sex marriage in Nepal contrasts sharply with the accepting later that he would have done precisely what PM Indira
disappointment felt in India following a Supreme Court Gandhi did in 1971—seize an opportune moment to break up
verdict that fell short of legalizing marriage equality. Pakistan and establish India’s primacy in the Subcontinent.
(c) The paradox lies in the fact that while India’s Supreme
After the Cold War ended in 1991 and Delhi embarked on
Court made strides toward LGBTQI+ rights, it stopped
economic reforms, Kissinger became an ardent supporter of the
short of legalizing marriage equality, whereas Nepal
achieved marriage equality through a landmark verdict. US-India partnership. After India’s nuclear tests in 1998, he was
among the first to urge Washington to recognise the reality of
(d) The passage introduces a paradox by portraying Nepal as
an atomic India and find a political accommodation with Delhi
the first South Asian nation to institutionalize marriage
by putting aside the non-proliferation ideology. In an interview
equality while India, its neighbouring country, struggles
earlier this year, Kissinger said India’s foreign policy under
to make significant progress in LGBTQI+ rights.
PM Narendra Modi comes closest to his statecraft based on
15. What assumption underlies the hope expressed in the realpolitik. Kissinger’s praise is not surprising, given the profound
passage regarding the impact of Nepal’s same-sex marriage reorientation of Indian diplomacy in recent years. Yet, Delhi also
legalization on other countries? needs to learn from the tragedy of Kissinger’s worldview. While
(a) The assumption is that the legal recognition of same- he sought to nudge the US towards pragmatism and away from the
sex marriage in Nepal will automatically inspire other twin extremes of isolationism and ideological crusades, he had to
countries, especially in South Asia, to follow suit. repeatedly sacrifice the moral imperatives of justice in the pursuit
(b) The assumption is that the recognition of same-sex of order and stability—from Indo-China to South Asia and Latin
marriage in Nepal is solely based on the influence of America. That made him a hated figure within left-liberal circles
global ideas and not on the country’s internal socio- in the US and beyond. Delhi may be right to reorient its foreign
cultural factors. policy towards realism, but it can’t afford to lose the balance
(c) The assumption is that the disappointment felt in India between power and principle, between interests and values, in the
following its Supreme Court verdict will prompt the conduct of its international relations.
country to quickly adopt the same-sex marriage model Source: Express View: Henry Kissinger was a man of the
from Nepal. world | The Indian Express
(d) The assumption is that the legal decisions in Nepal have
16. What structural pattern does the passage primarily follow,
always set an example for its neighbouring countries to
follow in matters of social and cultural change. and what does it suggest about the author’s perspective on
Henry Kissinger’s legacy?
PASSAGE 4 (a) The passage begins with an overview of Kissinger’s
Henry Kissinger, the best-known American diplomat of modern controversial reputation in India, followed by an
times, is deeply associated in the Indian mind with the American examination of specific historical events, and concludes
tilt to Pakistan during the 1971 war to liberate Bangladesh. Many with a cautionary note for India’s foreign policy.

Practice Questions 49
(b) The passage starts with a critical assessment of (c) There is a contradiction in the passage regarding
Kissinger’s role in the 1971 war, then delves into his Kissinger’s stance on India, as it initially portrays him
contributions to shaping American foreign policy, and negatively but later praises his support for the U.S.-India
ends with a reflection on the lessons for India’s rising partnership.
role in the world. (d) The passage doesn’t adequately address the criticism
(c) The passage unfolds chronologically, starting with that Kissinger’s realpolitik often compromised moral
Kissinger’s actions during the 1971 war, moving to the imperatives for the sake of stability.
post-Cold War era, and concluding with a caution against 20. Which of the following scenarios would most parallel the
India following Kissinger’s worldview. realpolitik approach of Henry Kissinger, as described in the
(d) It begins with India’s perception of Kissinger, transitions passage?
to his contributions, then critiques his realpolitik (a) A country, in pursuit of economic interests, forms
worldview, and finally suggests lessons for India’s
strategic alliances with nations known for human rights
foreign policy.
violations, overlooking ethical concerns.
17. What can be reasonably inferred about Henry Kissinger’s (b) A nation adopts an isolationist stance, avoiding
evolving stance towards India based on the passage? interference in global affairs, to focus on domestic
(a) Kissinger’s early Cold War-era isolationist approach led economic growth and stability.
to a strained relationship with India, but he later became
(c) A government, prioritizing moral imperatives,
a strong advocate for a robust India-U.S. partnership.
actively engages in international conflicts to address
(b) Kissinger consistently viewed India as a Cold War ally humanitarian issues, even at the risk of strained relations.
and remained critical of its actions, particularly during
(d) A country seeks ideological alignment with multiple
the 1971 war, irrespective of changing geopolitical
nations to build a coalition against a common adversary,
dynamics.
emphasizing shared values over pragmatic interests.
(c) Kissinger’s initial attempts to isolate China were
opposed by India in the 1950s, but later, he recognized PASSAGE 5
the importance of a strategic partnership with India in
Sometimes, all that is needed is someone who will listen. In the
the post-Cold War era.
cities of central and west Africa, among the poorest regions of the
(d) The passage provides no information about Kissinger’s world, where mental health support is beyond access for many,
evolving stance towards India; it primarily focuses on hairdressers are being trained to provide at-risk people with a
his legacy and contributions. sympathetic ear and a shoulder to cry on. Designated as “mental
18. What could potentially weaken the argument that Henry health ambassadors”, they are taught by mental health professionals
Kissinger’s statecraft holds valuable lessons for a rising to spot non-verbal signs of distress as they braid and blow-dry their
India? clients’ hair, gently seek information and give advice.
(a) Kissinger’s controversial legacy, including his sacrifice This creative approach to addressing what is now recognised
of moral imperatives for stability, may be viewed as a as a basic need has a lesson for other countries struggling with
liability rather than a source of valuable lessons. a similar combination of low awareness and limited access to
(b) The passage emphasizes Kissinger’s support for Pakistan infrastructure and trained personnel. In India, for example, as
during the 1971 war, which could be considered a noted by the Parliamentary Standing Committee on Health and
questionable approach from an ethical standpoint. Family Welfare in August, there are only 0.75 psychiatrists per
(c) Kissinger’s praise for India’s foreign policy under 1,00,000 population. The Union government’s national tele-
PM Narendra Modi contradicts the argument that his mental health programme, called Tele Mental Health Assistance
statecraft provides relevant lessons for a rising India. and Networking Across States (Tele-MANAS), providing free
(d) The passage suggests that Kissinger’s realpolitik and round-the-clock assistance to those in distress, has made
approach was more suitable for the Cold War era, raising some difference—in his address at a Mental Health Day event last
doubts about its applicability to the present geopolitical month, the Union Minister for Health said that 3,50,000 people
landscape. had been counselled since the initiative’s rollout in 2022. But in
19. What is the central flaw in the argument that Henry a nation of 1.4 billion, where about 40 per cent of the population
Kissinger’s statecraft holds valuable lessons for a rising has reported mental health distress, more needs to be done.
India? This could mean more proactive measures, including training
(a) The argument fails to consider Kissinger’s early attempts people whose work involves daily contact with others, to recognise
to isolate Communist China when India championed its signs of distress. Salon professionals are particularly well-suited
cause in the 1950s. for this, as seen in the African initiative, as well as programmes
like Hair 3Rs and Hairdressers with Hearts in Australia. As long
(b) The passage downplays the impact of the U.S. tilt
as shame, lack of knowledge and money remain deterrents,
towards Pakistan during the 1971 war on India,
creating networks of support within communities and enabling
overlooking potential consequences.

50 Critical Reasoning PW
conversations in intimate, non-intimidating settings, can help a lot (c) The shortage of mental health professionals in India
more people in distress. necessitates proactive measures.
Source: Blowdry and vent: Now, hairdressers who are (d) Initiatives like Hair 3Rs in Australia can be implemented
qualified to counsel | The Indian Express successfully in India.
21. What is the primary purpose of the passage? 24. Which of the following presents a paradox discussed in the
(a) To highlight the shortage of mental health professionals passage?
in India and criticize the government’s efforts in (a) The national mental health program in India has made a
addressing the issue. significant impact despite the shortage of psychiatrists.
(b) To showcase an innovative approach in central and west (b) In central and west Africa, mental health support is
Africa where hairdressers are trained to support people recognized as a basic need, but access to infrastructure
with mental health issues. and trained personnel is limited.
(c) To commend the success of India’s national mental health (c) Hairdressers in Africa are trained as mental health
program and its positive impact on the population. ambassadors, addressing the lack of awareness and
(d) To emphasize the need for more mental health awareness access to support.
and support initiatives, especially in countries with
(d) Despite 40% of the Indian population reporting mental
limited access to mental health infrastructure.
health distress, only 0.75 psychiatrists per 1,00,000
22. What role does the statement “In India, for example, as noted population are available.
by the Parliamentary Standing Committee on Health and
Family Welfare in August, there are only 0.75 psychiatrists 25. Which of the following, if true, would most weaken the
per 1,00,000 population” play in the passage? argument presented in the passage?
(a) To criticize the efforts of the Indian government in (a) The national mental health program in India has received
addressing mental health issues. positive feedback from a majority of those who have
(b) To emphasize the success of India’s mental health undergone counselling.
initiatives. (b) Mental health awareness campaigns in India have led to
(c) To provide statistical evidence supporting the need an increase in the number of people seeking professional
for proactive measures in addressing mental health help for mental health issues.
challenges in India. (c) The training of hairdressers as mental health ambassadors
(d) To highlight the shortage of mental health professionals in Africa has not resulted in a significant improvement
in central and West Africa. in mental health outcomes for at-risk individuals.
23 What is the main conclusion drawn from the passage? (d) In India, the majority of mental health distress cases are
(a) Hairdressers in central and west Africa are providing related to economic factors, and addressing these factors
effective mental health support. would have a more substantial impact on overall mental
(b) The Indian government’s Tele-MANAS program has well-being.
significantly improved mental health assistance.

Answer Key

1. (a) 2. (b) 3. (d) 4. (b) 5. (c) 6. (c) 7. (c) 8. (d) 9. (c) 10. (d)
11. (b) 12. (b) 13. (b) 14. (b) 15. (d) 16. (b) 17. (c) 18. (a) 19. (b) 20. (b)
21. (b) 22. (c) 23. (c) 24. (d) 25. (d)

Practice Questions 51
Solutions

1. (a) Correct. The passage primarily revolves around the the TC-5 meeting.
operationalization of the Loss and Damage (L&D) fund Option (c) Incorrect. The passage mentions that committed
and highlights the challenges and concerns associated with amounts, including those by Japan, Germany, and the UAE,
its implementation. It discusses the host organization, the are insufficient, and it does not suggest that developing
commitments made by some countries, and the conditions countries deem them sufficient.
set for fund management. Option (d) Correct. The passage indicates that issues in
Option (b) Incorrect. The passage does mention the the TC-4 meeting spilt over into an ad hoc TC-5 meeting,
commitments made by some countries, but it doesn’t highlighting the crucial role of both meetings in determining
emphasize them as the main theme. Instead, it discusses the the operationalization of the L&D fund.
insufficiency of the committed amounts and the voluntary
4. (a) Incorrect. This option highlights a factual situation where
nature of contributions.
committed amounts are insufficient, and some developing
Option (c) Incorrect. While the passage mentions the COP28 countries initially resisted financial contributions, but it
climate talks and the diplomatic victory of its Emirati doesn’t present a paradox.
president, these are not the central themes. The primary
Option (b) Correct. The hosting of the L&D fund by the
focus is on the challenges and issues related to the newly
World Bank, expected to charge a significant overhead fee,
operationalized L&D fund.
creates a paradox where the overseeing institution poses a
Option (d) Incorrect. Although the passage discusses potential financial burden.
the World Bank’s role in hosting the L&D fund and the
Option (c) Incorrect. The delay caused by issues in the TC-4
conditions it needs to fulfil, this is not the central theme.
meeting and subsequent spillage into TC-5 is a chronological
The main theme is the operationalization of the L&D fund
sequence of events rather than a paradoxical situation.
and the challenges associated with it.
Option (d) Incorrect. The missed 2020 deadline for climate
2. (a) This statement is not an assumption but a stated concern finance commitments and subsequent commitments to the
in the passage. The passage mentions the insufficiency of L&D fund represent a factual situation without a paradox.
the committed amounts and questions whether they will be
periodically replenished. 5. (a) Incorrect. While insufficient commitment amounts are
discussed in the passage, it is not considered a flaw but a
Option (b) Correct. The concerns in the passage revolve
factual situation.
around the assumption that the World Bank’s stewardship
of the L&D fund may face challenges and conditions related Option (b) Incorrect. The initial resistance of developing
to transparency may not be adequately met. The passage countries to financial contributions is not presented as a
discusses the conditions imposed on the World Bank and central flaw but as part of the negotiation process.
the potential challenges associated with its role. Option (c) Correct. The central flaw lies in the World Bank’s
Option (c) Incorrect. The passage does not assume that oversight, with the expectation of a significant overhead fee,
COP28 climate talks were unsuccessful. It mentions the creating a potential financial burden on developing countries.
success of the talks in agreeing to operationalize the L&D Option (d) Incorrect. Voluntary contributions are mentioned
fund. in the passage but are not highlighted as a central flaw in
Option (d) Incorrect. The passage acknowledges the role of the operationalization of the L&D fund.
the G-77 bloc of countries plus China in the launch of the 6. (c) Correct. The passage follows a structure where it
L&D fund during the COP27 talks. It doesn’t assume their introduces GDP growth estimates, discusses the performance
negligible role. of key sectors, analyzes the slowdown in private consumption,
3. (a) Incorrect. The passage mentions concerns about the and concludes with a call for inclusive growth.
L&D fund’s operationalization, but it doesn’t specify that Option (a) Incorrect. While this option captures some
developing countries were primarily concerned about elements, it does not specifically mention the analysis of
financial commitments. The concerns include the hosting of the slowdown in private consumption, a crucial aspect of
the fund by the World Bank and the insufficient committed the passage.
amounts. Option (b) Incorrect. This option misrepresents the order of
Option (b) Incorrect. The passage states that the L&D fund the topics discussed in the passage.
will be hosted by the World Bank for an interim period, and Option (d) Incorrect. While this option includes elements
developing countries resisted this proposition initially but from the passage, it does not accurately reflect the order in
yielded later. It does not imply successful resistance during which the topics are presented.

52 Critical Reasoning PW
7. (c) Correct. The passage explicitly mentions that the recent Option (a) Incorrect. This scenario involves a surge in profits
economic momentum owes its fillip in large measure for a technology company, but it does not mirror the decline
to front-loaded government spending, both in terms of and subsequent growth pattern described in the passage.
consumption demand and asset-creating capital investments. Option (b) Incorrect. While it involves a decline in sales for
Option (a) Incorrect. The passage does not support the claim a retail chain, the passage discusses overall growth in the
that robust private consumption spending is the primary manufacturing sector, which is not paralleled in this option.
driver of the recent economic momentum. Option (c) Incorrect. The scenario of a pharmaceutical
Option (b) Incorrect. While the slowdown in the growth of company achieving record-breaking sales does not parallel
the services sector is discussed, the passage does not suggest the growth pattern discussed for the manufacturing sector
that the growth in this sector is the primary factor behind in the passage.
the recent economic momentum. 11. (b) Correct. The passage discusses Nepal’s Supreme Court
Option (d) Incorrect. The passage does not provide legalizing same-sex marriage and how this development
information to support the claim that the expansion in the has the potential to inspire positive change in the region,
financial and realty services sector has been the key driver including India, where recent legal developments regarding
of the recent economic momentum. LGBTQI+ rights were not as progressive as hoped.
8. (d) Correct. This statement directly supports the argument Option (a) Incorrect. While this option mentions the Supreme
by highlighting a potential challenge to the recent economic Court’s decision in India, it does not capture the broader
momentum—the fact that rural demand remains affected theme of the passage, which emphasizes the influence of
by below-average monsoon conditions. This suggests a Nepal’s decision on the region.
limitation in the broad-based growth. Option (c) Incorrect. While Maya Gurung and Surendra
Option (a) Incorrect. This option suggests a potential Pandey’s wedding is mentioned in the passage, the main idea
continuation of government spending, which, if anything, is not solely focused on this specific event but rather on the
strengthens the argument rather than indicating a challenge. broader implications for LGBTQI+ rights in the region.
Option (b) Incorrect. This option provides a positive outlook Option (d) Incorrect. While the interconnected world and
for private final consumption expenditure, which weakens the power of influential ideas are discussed in the passage,
the argument about potential challenges to economic the central theme revolves around Nepal’s legalization of
same-sex marriage and its potential impact on the region,
momentum.
particularly India.
Option (c) Incorrect. The anticipated robust pace of growth
in the manufacturing sector does not address the concerns 12. (b) Correct. This option weakens the argument by
or challenges mentioned in the argument, and therefore, it highlighting the scepticism expressed by LGBTQI+
does not strengthen the argument. activists in India about drawing inspiration from legal
developments in neighbouring countries, suggesting that
9. (c) Correct. The passage states that construction witnessed such developments may not necessarily lead to positive
its best showing in five quarters, which indicates positive change.
performance driven by a favourable base effect.
Option (a) Incorrect. Opposition from conservative religious
Option (a) Incorrect. The passage mentions that the groups within Nepal does not necessarily undermine the
manufacturing sector was the strongest performer in the argument about inspiring positive change in the broader
current quarter, growing at a nine-quarter high. region.
Option (b) Incorrect. The passage does not attribute Option (c) Incorrect. Social stigma and discrimination in
the decline in private final consumption expenditure to Nepal, despite legal recognition, may still be perceived as
government policies. a localized issue and not necessarily diminish the potential
Option (d) Incorrect. The passage does not provide impact on neighbouring countries.
information on a significant rebound in the services sector; it Option (d) Incorrect. The increased public discourse and
highlights concerns about the slowdown in key components awareness in India, prompted by the Supreme Court’s
of the services economy. decision, could be seen as a positive development that aligns
10. (d) Correct. The passage mentions that the manufacturing with the argument presented in the passage.
sector, driven by a favourable base effect, was the strongest 13. (b) Correct. The statement provides supporting evidence by
performer, achieving a nine-quarter high in growth. This is showcasing a positive outcome of Nepal’s legalization of
analogous to an automobile manufacturer facing a decline same-sex marriage. It reinforces the argument that legalizing
in production (manufacturing sector decline), but the same-sex marriage can lead to tangible and progressive
transportation industry as a whole sees increased demand changes.
for electric vehicles (overall growth in the manufacturing Option (a) Incorrect. The statement does not serve as a
sector). counterexample but rather as a positive illustration.

Practice Questions 53
Option (c) Incorrect. The statement does not present a 17. (a) Incorrect. The passage doesn’t suggest that Kissinger had
contradiction but aligns with the argument’s perspective on an isolationist approach during the Cold War. It highlights
the positive impact of legal recognition. his realpolitik, which involved both protecting allies and
Option (d) Incorrect. The statement actively contributes to forming strategic partnerships.
the argument by presenting a concrete example of a positive Option (b) Incorrect. The passage acknowledges Kissinger’s
development resulting from the legalization of same-sex role in protecting Pakistan during the 1971 war but also notes
marriage. his support for the U.S.-India partnership post-Cold War.
14. (b) Correct. The paradox is evident in the contrast between Option (c) Correct. The passage indicates that after the
the positive development of Nepal legalizing same-sex Cold War, Kissinger became a supporter of the U.S.-India
marriage and the disappointment expressed in India partnership, especially after India’s nuclear tests in 1998.
following a Supreme Court verdict that did not fully embrace Option (d) Incorrect. The passage does provide information
marriage equality. about Kissinger’s evolving stance towards India, especially
Option (a) Incorrect. The values in Nepal are not described in the post-Cold War period.
as paradoxical in the passage. 18. (a) Correct. The argument in the passage revolves around
Option (c) Incorrect. While there is a comparison between Kissinger’s statecraft providing lessons for a rising India. If
India and Nepal, the paradox lies more in the contrast readers perceive Kissinger’s sacrifice of moral imperatives
between the two countries’ outcomes rather than in the as a significant drawback, it weakens the argument.
actions of India’s Supreme Court. Option (b) Incorrect. The passage acknowledges Kissinger’s
Option (d) Incorrect. The paradox is not solely based support for Pakistan during the 1971 war but doesn’t
on the comparison between Nepal and India but on the explicitly argue that it weakens the lessons for a rising India.
unexpected contrast between Nepal’s achievement and Option (c) Incorrect. Kissinger’s praise for India’s foreign
India’s disappointment. policy under PM Narendra Modi aligns with the argument,
15. (a) Incorrect. While the passage suggests hope for a positive reinforcing the idea that his statecraft offers valuable lessons.
impact, it doesn’t explicitly state that other countries will Option (d) Incorrect. While the passage mentions the end
automatically follow Nepal’s lead. of the Cold War, it doesn’t explicitly argue that Kissinger’s
Option (b) Incorrect. The passage doesn’t exclusively attribute realpolitik is outdated or unsuitable for the present
Nepal’s decision to global influence; it acknowledges a geopolitical landscape.
model being available in the neighbourhood. 19. (a) Incorrect. The argument doesn’t necessarily need to
Option (c) Incorrect. The passage does not make this consider every aspect of Kissinger’s career, and the focus
assumption; it focuses more on the positive influence of is on his contributions and lessons for a rising India.
Nepal’s example rather than disappointment in India. Option (b) Correct. The central flaw is that the argument
Option (d) Correct. The assumption is that Nepal’s decisions, downplays or dismisses the impact of the U.S. tilt towards
especially regarding social and cultural change, have the Pakistan during the 1971 war, which could be considered a
potential to influence neighbouring countries, as suggested significant flaw in Kissinger’s statecraft.
by the reference to the RTI law. Option (c) Incorrect. The passage does not portray Kissinger
16. (a) Incorrect. The passage doesn’t follow this structure. It negatively but rather acknowledges his evolving stance and
provides a broader perspective on Kissinger’s legacy and praises his support for the U.S.-India partnership.
lessons for India, rather than focusing on specific historical Option (d) Incorrect. The passage acknowledges Kissinger’s
events. sacrifice of moral imperatives but does not consider it a flaw
Option (b) Correct. The passage begins by addressing in the argument.
Kissinger’s association with India and China, then explores 20. (a) Correct. This scenario aligns with the realpolitik
his contributions, and concludes by reflecting on the approach, where economic interests take precedence over
implications for India’s foreign policy. ethical concerns, reflecting Kissinger’s pragmatic stance.
Option (c) Incorrect. The passage does not strictly follow Option (b) Incorrect. This option describes an isolationist
a chronological order. It discusses various aspects of stance, which contrasts with Kissinger’s approach to active
Kissinger’s legacy without adhering to a strict timeline. international engagement.
Option (d) Incorrect. While the passage does discuss India’s Option (c) Incorrect. This option emphasizes moral
perception, Kissinger’s contributions, and lessons for India, imperatives, which go against the realpolitik approach
it does not strictly follow this sequence. advocated by Kissinger.
This question assesses the reader’s understanding of the Option (d) Incorrect. This scenario emphasizes shared
structural organization of the passage and its implications values over pragmatic interests, which is not consistent with
for the author’s perspective on Kissinger’s legacy. Kissinger’s realpolitik.

54 Critical Reasoning PW
21. (b) Correct. The passage primarily discusses the innovative the broader need for proactive measures due to the shortage
approach of training hairdressers in central and west Africa of mental health professionals.
to provide mental health support, drawing attention to the Option (d) Incorrect. The passage does not specifically
potential application of similar methods in other countries. discuss the implementation of initiatives like Hair 3Rs in
Option (a) Incorrect. While the passage mentions the Australia in India.
shortage of psychiatrists in India, it does not criticize the 24. (d) Correct. The paradox discussed in the passage is the
government’s efforts but rather suggests a creative approach contrast between the high reported mental health distress
to address the issue. in India (40% of the population) and the low availability of
Option (c) Incorrect. Although the passage mentions psychiatrists (0.75 per 1,00,000 population). This creates a
India’s tele-mental health program, it does not focus on paradoxical situation where there is a significant need for
commending its success but rather highlights the need for mental health assistance, but the professional resources are
more proactive measures. insufficient.
Option (d) Incorrect. While the passage emphasizes the Option (a) Incorrect. This option does not present a paradox.
need for mental health awareness and support initiatives, It states that the national mental health program has made
its primary focus is on the innovative approach involving a significant impact, which is not contradictory within the
hairdressers in Africa. context of the passage.
22. (c) Correct. The statement is used to provide statistical Option (b) Incorrect. While the limited access to mental
evidence supporting the need for proactive measures in health support in central and west Africa is highlighted, it
addressing mental health challenges in India. It contributes doesn’t present a paradox within the passage.
to the argument that there is a shortage of mental health Option (c) Incorrect. The training of hairdressers as mental
professionals in the country. health ambassadors in Africa is presented as a creative
Option (a) Incorrect. The statement does not criticize the solution, but it doesn’t introduce a paradox in the context
efforts of the Indian government; instead, it focuses on of the passage.
presenting a factual observation. 25. (d) Correct. This option would weaken the argument by
Option (b) Incorrect. The statement does not emphasize the suggesting that addressing economic factors, rather than
success of India’s mental health initiatives but rather points solely focusing on increasing the number of psychiatrists
out a challenge. or mental health support programs, could have a more
Option (d) Incorrect. The statement is specifically about significant impact on mental well-being in India.
India and its mental health professional shortage, not central Option (a) Incorrect. Positive feedback from those who
and west Africa. underwent counselling does not necessarily weaken the
23. (c) Correct. The main conclusion of the passage is that the argument; it could support the effectiveness of the mental
shortage of mental health professionals in India necessitates health program.
proactive measures. The passage discusses the training of Option (b) Incorrect. An increase in people seeking
hairdressers in Africa as a creative approach and suggests professional help due to mental health awareness campaigns
that similar proactive measures are needed in India. does not directly weaken the argument. It could be seen as
Option (a) Incorrect. The passage does discuss the use of a positive outcome.
hairdressers in Africa but does not make a general claim Option (c) Incorrect. The lack of significant improvement
about their overall effectiveness in providing mental health in mental health outcomes from training hairdressers as
support. mental health ambassadors does not necessarily weaken the
Option (b) Incorrect. While the Tele-MANAS program is argument. The effectiveness of this specific program is not
mentioned as making some difference, the main focus is on the main focus of the passage.

Practice Questions 55
CLAT ACHIEVERS
Pro 2025
For CLAT, AILET & OLETs

Daily Practice Problems With


Video Solutions
Monthly Magazines of Current Affairs
& Legal Reasoning (January 2024
to April 2025)
10,000+ Questions Bank in DPP Format &
Lecture Notes

30 Study E-Modules & Practice


E-Modules
Annual subscription to access all the
upcoming batches of CLAT 2025

Khazana Package: 120+ Best Lectures


of CLAT 2024
80+ Hours Marathon Revision Lectures
(Oct-Nov. 2024)

30 Mock Test Series with video solutions

10,000+ Infinite Practice Questions


(Passage Based)

SAVE BIG on CLAT ACHIEVERS Pro 2025 Enrolment

Enjoy a Discount of FLAT ₹ 500/- Off with COUPON CODE CLAT500

You might also like